SlideShare ist ein Scribd-Unternehmen logo
1 von 28
Plants and Creatures of Dermatologic Significance
1) Name this creature:

A. Demodex folliculorum

B. Ornithonyssus sylviarum

C. Pediculus capitis

D. Pediculus humanusCorrect Choice

E. Pthirus pubis
This is the body louse, or Pediculus humanus var. corporis. The pubic louse (pthirus pubis) is
shorter, and squatter. The head louse (pediculus capitis) has a narrow body


2) When doing patch testing for compositae sensitivity, which of the following substances is used?

A. Tuliposide A

B. Rhus

C. Sesquiterpene lactone mix Correct Choice

D. Colophony

E. Pentadecacatechols
Colophony cross reacts with turpentine resin, hairpiece adhesive, mascara, dental cement and
others. Pentadecacatechols are in the rhus family and are found in poision ivy, oak and sumac in
addition to cross reacting with mango and cashew. Tuliposide A is found in Peruvian lily
(alstomeria), and tulip


3) A previously healthy 25-year-old man presents to the ER 8 hours after experiencing an
extremely painful spider bite. The bite site is erythematous and indurated, without necrosis or
eschar. The patient is vomitting and experiencing chills, violent cramps, and abdominal pain and
rigidity. The surgical consultant suspects an acute abdomen. You correctly propose that:


A. The patient has a likely Lycosidae bite and needs supportive care

B. The patient has a likely Loxosceles bite and should go to the OR for excision and debridement

C. The patient needs anti-venom after a likely Phidippus formosus bite

D. The patient has likely experienced a hobo spider bite

E. The patient needs anti-venom after a likely Lactrodectus biteCorrect Choice
The patient has signs and symptoms a black widow spider bite, caused by Lactrodectus. Anti-venom
may be helpful up to 90 hours after the bite


4) Match the photo to the plant identified:

A. Poison oak

B. Poison sumac Correct Choice

C. Mango leaves

D. Gingko

E. Poison ivy
Poison sumac leaves grow in groupings of 7-13 with a single leaf at the tip.



                                                  1
5) All of the following are food mites except:

A. DermatophagoidesCorrect Choice

B. Acarus

C. Glyciphagus

D. All of the listed answers are food mites

E. Tyrophagus
Acarus (grain mite), Tyrophagus (grocery mite), and Glyciphagus (cheese mite) are all food mites
that ingest foodstuffs. They can produce papular urticaria or vesicopapular eruptions.
Dermatophagoides is the dust mite, and causes environmental allergic reactions.


6) Which of the following statements regarding Megalopyge opercularis is correct?

A. Are also called "bed bugs" by lay people

B. These creatures can be vectors of many diseases from typhus to plague

C. These creatures, found in Hawaii, are nocturnal carnivores that produce painful wounds by
discharging venom

D. Contact with this creature often causes immediate pain, urticaria, and erythematous papules or
purpura in a tram-track patternCorrect Choice

E. Bites are usually asymptomatic, initially, but then form linear purpuric macules
Megalopyge opercularis (puss caterpillar) is one of the most widespread causes of caterpillar
dermatitis. Contact with this creature often causes immediate pain, urticaria, and erythematous
papules or purpura in a tram-track pattern. Bites of Cimicidae (bedbugs) are usually asymptomatic,
initially, but then form linear purpuric macules. Fleas can be vectors of many diseases from typhus
to plague. Chilopoda (centipedes) are nocturnal carnivores that produce painful wounds by
discharging venom. Scolopendra is a species of centipede found in Hawaii


7) The venom from which of the following creatures contains phospholipase A?

A. Cimicidae

B. Ctenocephalides canis

C. Theraphosidae

D. HoneybeeCorrect Choice

E. Solenopsis invicta
Honeybees (Hymenoptera) leave a barbed ovipositor and paired venom sacs impaled into their
victim. The honeybee dies after stinging, but other hymenoptera do not. The venom from the
honeybee contains phospholipase A. Theraphosidae (tarantulas) are of dermatologic importance due
to the presence of urticating hairs. Cimicidae (bedbugs) cause initially asymptomatic bites that then
form linear, purpuric, pruritic macules and erythematous papules. Ctenocephalides canis is a flea
that causes frequent infestations on dogs. Solenopsis invicta is the imported fire ant, and attacks in
groups. Solenopsis inject a venom that contains a hemolytic factor, solenopsin D


8) Unilateral swelling of the eyelid at the site of the bite of the assassin bug is called what?

A. Rothschild's sign




                                                    2
B. Romana's signCorrect Choice

C. Russell's sign

D. Raynaud's sign

E. Unilateral Chagas' disease
Rothschild's sign is alopecia of the lateral 1/3 of the eyebrows secondary to hypothyroidism.
Russell's sign is the development of callouses on the dorsal dominant hand. Raynaud's sign is the
development of acrocyanosis


9) A butcher’s nodule is caused by which of the following?

A. HPV 2, 7 Correct Choice

B. HPV 16, 18

C. HPV 1

D. HPV 24

E. HPV 13, 32
HPV 24 causes common warts. HPV 16 and 18 are found in genital warts; 1 is for plantar warts and
types 13 and 32 are seen in Heck’s Disease (focal epithelial hyperplasia


10) Immunologic contact urticaria is frequently caused by:

A. None of these answers are correct

B. Plants of the Urticaceae family

C. Hevea brasiliensisCorrect Choice

D. All of these answers are correct

E. Plants of the Euphorbiacea family
Contact urticaria occurs after direct contact, and can be immunologic (IgE) or non-immunologic
(toxin-mediated). Type I hypersensitivity, or IgE mediated reactions, require previous sensitization.
Frequent causes of immunologic eruptions include latex proteins (Hevea brasiliensis). Plants such as
stinging nettles (Urticaceae family) and spurge nettle (Euphorbiaceas) cause nonimmunologic
urticaria, which occurs without previous sensitization in any host.


11) Phytophotodermatitis is caused by which of the following wavelengths of light?

A. 290-320 nm

B. 320-400 nm Correct Choice

C. 100-200 nm

D. 200-290 nm

E. 311 nm
UVA rays are responsible for the phototoxic reaction which occurs with phytophotodermatitis


12) Identify this caterpillar:

A. Greenback caterpillar

B. Hag moth caterpillar



                                                 3
C. Io caterpillar

D. Puss caterpillar

E. Saddleback caterpillarCorrect Choice
This is the saddleback caterpillar, as it appears that the caterpillar is wearing a saddle


13) Which of the following is false regarding this image?

A. This is a glossina fly

B. This transmits trypanosomes

C. A large abscess may occur following a biteCorrect Choice

D. This transmits sleeping sickness

E. This is the tsetse fly
Cutaneous reactions tend to be minimal from the bite of the tsetse, or glossina, fly which transmits
trypanosomes and sleeping sickness


14) Flying squirrels can carry many infections, including which of the following?

A. all of these infections are correctCorrect Choice

B. Toxoplasma gondii

C. Staphlyococcus sp

D. none of these infections are correct

E. Epidemic typhus
Flying squirrels can carry many infections, including Toxoplasma gondii, Staphylococcus sp., and
Rickettsia prowazekii (via the body lous, causing epidemic typhus


15) A patient who experiences an allergic contact dermatitis in reaction to an extract from the Pinus
palustris tree may also be sensitized to:


A. None of these answers are correct

B. Varnish

C. Mascara

D. Rosin

E. All of these answers are correctCorrect Choice
Colophony, derived from the Pinus palustris tree, may cross react with rosin, medications, mascara,
adhesives, bandages, varnish, wax, paper products, and dental cement


16) A butcher presents with numerous verrucous papules of the hands and fingers. Based on her
occupation, you consider a diagnosis of infection with which HPV types?


A. HPV 2 and 7Correct Choice

B. HPV 1 and 2

C. HPV 6 and 8

D. HPV 16 and 18




                                                    4
E. HPV is not associated with this condition
Meat, fish, and poultry handlers often suffer from hand and finger verrucae, usually caused by
human papilloma virus types 2 and 7.


17) This snake, the coral snake, is a member of what family?

A. ElapidaeCorrect Choice

B. Lepidoptera

C. Theraphosidae

D. Crotalidae

E. Apiaceae
The Crotalidae family includes rattlesnakes and copperheads. Lepidoptera are caterpillars.
Theraphosidae is the scientific name for tarantulas. Apiaceae (Umbelliferae) is a plant family which
includes parsley, celery and fennel. Elapidae is the correct family of snakes, including coral snakes


18) Which of the following plants contains thiocyanates that can cause an irritant dermatitis?

A. Radish

B. All of these answers are correctCorrect Choice

C. Plants of the Brassicaceae family

D. Mustard

E. Garlic
Thiocyanates are found in garlic (Alliaceae family), mustard, and radish (Brassicaceae family


19) This tick transmits:

A. Babesiosis

B. Erlichiosis

C. Rocky mountain spotted feverCorrect Choice

D. Relapsing fever

E. Lyme
This is Dermacentor variabilis which transmits Rocky Mountain Spotted Fever as well as Tularemia


20) Which family of plants is most commonly implicated in phytophotodermatitis caused by
furocoumarins?


A. Moraceae

B. Papilonaceae

C. ApiaceaeCorrect Choice

D. Compositae

E. Rutaceae
The Apiaceae (formerly umbelliferae) family is the most common cause of phytophotodermatitis.
The members of this family include cow parsley, celery, wild parsnip, false bishop's weed, giant
hogweed, angelica, meadow grass, fennel, wild carrot, caraway, and coriander



                                                    5
21) The reaction caused by this plant is mediated by:

A. IgE

B. Eosinophils

C. Langerhans cellsCorrect Choice

D. Mast cells

E. Histamine
This is ragweed, which causes allergic contact dermatitis. Mast cells, IgE and histamine are
characteristic of contact urticaria. Allergic contact dermatitis is caused by langerhans cells
presenting antigen to CD4+ lymphocytes leading to a cell-mediated delayed hypersentitivity
reaction


22) Which of the following mites is a cause of "walking dandruff" in dogs and cats?

A. All of these answers are correct

B. None of these answers are correct

C. Allodermanyssus sanguineus

D. Acarus

E. CheyletiellaCorrect Choice
Cheyletiella mites are harbored by dogs and cats and cause walking dandruff. The pet is
asymptomatic, but people handling the pet may experience pruritus when mites feed on skin.
Diagnosis is by microscopic examination of cellophane tape applied to the pet's skin. Acarus is the
grain mite, and causes baker's itch. Allodermanyssus sanguineus is the house mouse mite, and the
vector of rickettsial pox


23) Which of the following statements regarding Theraphosidae is correct?

A. All of these statements are correctCorrect Choice

B. These creatures possess urticating hairs that are of dermatologic significance

C. These creatures can cause visual loss by causing a chronic granulomatous reaction (ophthalmia
nodosa)

D. None of these statements are correct

E. Bites do not generally produce any systemic toxicity
Theraphosidae (tarantulas) are large, brown to black, hairy spiders frequently found in the
southwest. They are of dermatologic importance due to urticating hairs. Tarantula bites generally do
not produce any systemic toxicity. The spider releases hairs in the direction of the perceived
attacker; hairs can penetrate the skin as deeply as the reticular dermis. If they injure the cornea,
they can cause a chronic granulomatous reaction (ophthalmia nodosa) and loss of vision


24) A veterinarian presents with an edematous pustule on the right arm, with nodules along the
lymphatic drainage route. You consider a diagnosis of glanders. What treatment do you
recommend?


A. Surgical excision followed by antibiotic treatment with streptomycin combined with
tetracyclineCorrect Choice




                                                  6
B. Reassurance. No treatment is necessary, as infection is self-limited and should resolve within 6
weeks

C. Surgical excision followed by antibiotic treatment with rifampin and ethambutol

D. Surgical excision

E. Antibiotic treatment with streptomycin combined with tetracycline
Farcy, or glanders, is caused by infection with the gram-negative rod Pseudomonas mallei. It most
frequently infects horses, donkeys, and mules; humans can occasionally also be infected. An
edematous nodule, pustule, or vesicle develops at the inoculation site. Nodules are often present
along the lymphatic drainage route and are called "farcy buds." Importantly, nasal ulceration and
septum perforation can be caused by glanders. A chronic form leads to deep, painful abscesses.
Treatment is with surgical excision of the lesion followed by streptomycin combined with
tetracycline.


25) Ananas comosus causes an irritant dermatitis attributed to which irritant?

A. Phorbol esters

B. Thiocyanates

C. Capsaicin

D. Ranunculin

E. BromelinCorrect Choice
Ananas comosus is the pineapple. Pineapples contain the irritants bromelin (a proteolytic enzyme)
and calcium oxalate (which causes fissures that grant bromelin access to dermal vessels). Capsaicin
is found in chili peppers. Phorbol esters are found in spurges, crotons, poinsettas, and machineel
trees. Thiocyanates are found in garlic, mustard, and radishes. Ranunculin is found in buttercups


26) This is a:

A. Moth caterpillar

B. Woolybear caterpillarCorrect Choice

C. Brownbear caterpillar

D. Puss caterpillar

E. Furrybear caterpillar
This is a woolybear caterpillar


27) Onions and chives:

A. Are members of the family Anacardiacea

B. Contain diallyl disulfideCorrect Choice

C. Cross-react with artichoke in sensitized patients

D. Contain pentadecylcatechol

E. Are members of the same family as Gingko
Onions, garlic, and chive are members of the family Alliaceae and contain diallyl disulfide (also
allylpropyl disulfide, allicin


28) Poison sumac is:


                                                   7
A. A source of colophony

B. The common name for Myroxylon balsamum

C. A source of tuliposide A

D. The common name for Pinus palustris

E. A member of the Toxicodendron genusCorrect Choice
Poison ivy, oak, and sumac are members of the Anacardiacea family, Toxicodendron genus.
Alstromeria (Peruvian lily) is the source of tuliposide A. Myroxylon balsamum is the source of
Balsam of Peru. The Pinus palustris tree is the source of colophony


29) Cantharadin is derived from:

A. Ranunculaceae

B. Compositae

C. Lytta vesicatoriaCorrect Choice

D. Pentadecacatechol

E. Solanaceae
Ranunculaceae (buttercup) causes irritant dermatitis. Pentadecacatechol is the rhus antigen found
in poison ivy, oak and sumac. Solanaceae (chili pepper) contains capsaicin. Pyrethrin is derived
from compositae (chrysanthemum flower). Lytta vesicatoria (spanish fly) is a blister beetle from
which cantharadin is made


30) Which of these mites is the most likely cause of a pruritic papulovesicular eruption in a baker?

A. Dermanyssus

B. Dermatophagoides

C. none of the above

D. Cheyletiella

E. AcarusCorrect Choice
Acarus (grain mite) causes so-called "baker's itch." Dermatophagoides (dust mite) causes allergic
reactions, Dermanyssus (fowl mite) causes equine encephalitis, and Cheyletiella causes walking
dandruff in dogs and cats


31) Lipase is a notable active component of the venom of which of the following spiders?

A. Loxosceles

B. Hidippus

C. ChiracanthiumCorrect Choice

D. Lactrodectus

E. Lycosidae
The venom of Chiracanthium (sac spiders) contains lipase. The venom of Lycosidae (wolf spiders) is
notable for histamine; Lactrodectus (black widow spiders) for alpha lactrotoxin; Hidippus (jumping
spiders) for hyaluronidase; and Loxosceles (brown recluse spiders) for sphingomyelinase-D.


32) The reaction in this picture is most likely a result of:


                                                    8
A. UV exposure Correct Choice

B. Child abuse

C. Irritant response from garlic

D. Toxicodendron bullous reaction

E. Nickel
This is an example of phytophotodermatitis. This is a reaction between furocoumarins followed by
exposure to UVA


33) Furocoumarins, such as 5-methoxypsoralen or 8-methoxypsoralen:

A. Cause a type IV delayed hypersensitivity reaction after exposure to all wavelengths of UV light

B. Cause a phototoxic reaction after contact with the skin, followed by exposure to ultraviolet light in
the 320-400nm rangeCorrect Choice

C. Cause a phototoxic reaction after contact with the skin, followed by exposure to ultraviolet light in
the 280-320nm range

D. Cause a photoallergic reaction after contact with the skin, followed by exposure to ultraviolet light
in the 320-400nm range

E. Cause a photoallergic reaction after contact with the skin, followed by exposure to ultraviolet light
in the 280-320nm range
Unlike photoallergic reactions, which involve the immune system (type IV or delayed
hypersensitivity), phytophotodermatitis is non-immunologic. Photoallergic reactions are more
frequently caused by other, non-plant sources. Furocoumarins, such as 5-methoxypsoralen or 8-
methoxypsoralen, cause a phototoxic reaction after contact with the skin, followed by exposure to
ultraviolet light in the UVA range (320 to 400nm).


34) A 47 year old park ranger developed a diffuse, erythematous, eczematous eruption after
fighting a forest fire. What is the most likely allergen causing this airborn contact dermatitis?


A. Alphamethylene

B. Pyrethrins

C. D-usnic acidCorrect Choice

D. Psoralen

E. Diallyl disulfide
Lichen causes airborne allergic contact dermatitis in lumberjacks, forest workers, and people
exposed to firewood, funeral wreaths, and masculine fragrances. The primary allergen is d-usnic
acid


35) Which disease is transmitted by this insect?

A. Chagas diseaseCorrect Choice

B. Onchocerciasis

C. African sleeping sickness

D. Visceral leishmaniasis

E. Dracunculiasis




                                                   9
This a picture of the reduviid bug, also called the kissing bug or assassin bug. It is the vector for
American Trypanosomiasis (Chagas disease), which is caused by Trypanosoma cruzii. The reduviid
bug prefers to bite at mucocutaneous junctions


36) Which of the following plants are members of the Solanaceae family?

A. Prickly pear

B. Radish

C. Garlic

D. Crotons

E. Chili pepperCorrect Choice
Chili peppers belong to the Solanaceae family. Prickly pears (Opuntia vulgaris), crotons
(Euphorbiaceae family), garlic (Alliaceae family), and radishes (Brassicaceae family) are not
members of this family


37) Leishmania donovani is transmitted by:

A. Glossina

B. Callitroga americana

C. Attagenus megatoma

D. PhlebotomusCorrect Choice

E. Anopheles
Leishmania donovani is transmitted by Phlebotomid sandflies. Callitroga americana is the most
important cause of cutaneous myiasis in the United States. Glossina, the tsetse fly, transmits
sleeping sickness. Attagenus megatoma is a carpet beetle, the larvae of which can cause an allergic
contact dermatitis. Anopheles mosquitoes transmit malaria.


38) Sharp hairs on plants of the Urticaceae family contain which of the following toxins that are
released into the skin causing rapid edema, pruritus, and burning?


A. Serotonin

B. Histamine and acetylcholine

C. Acetylcholine

D. All of these answers are correctCorrect Choice

E. Histamine
Sharp hairs on plants such as stinging nettles (Urticaceae family) contain toxins (histamine,
serotonin, and acetylcholine) which are released into the skin causing rapid edema, pruritus, and
burning


39) Which of the following may NOT occur after contact with the organism in the photo?

A. Loss of vision

B. Purpura in a tram-track pattern

C. Urticaria

D. Pruritus




                                                    10
E. Tram-track calcification on skull x-ray Correct Choice
The most common response is erythema, pruritus and urticaria. Purpura and hemorrhage in a tram-
track or ladder pattern may be seen as well. Ophthalmia nodosa may occur if hairs injure the
cornea. Tram-track calcification on skull films is seen in Sturge-Weber syndrome


40) A patient comes to you with complaints of numerous erythematous pruritic papules under her
bathing suit after swimming in the ocean. What is the most likely diagnosis?


A. Seabather’s eruption Correct Choice

B. Swimmer’s itch

C. Nematocyst stings

D. Hydrozoa contact

E. Hot tub folliculitis
Sea bather’s eruption is seen after ocean exposure and can be intensely pruritic. It is due to jellyfish
larvae (Linuche unguiculata) and can be prevented by rinsing off soon after coming out of the
water. Swimmer’s itch is a cercarial dermatitis caused by avian schistosomes which occurs after
freshwater exposure. It is usually seen on exposed areas. Portuguese man of war (hydrozoa) stings
can lead to hemorrhagic lesions with vesiculations


41) Which of the following plant families commonly causes phytophotodermatitis?

A. Rubiaceae

B. Ranunculaceae

C. Apiaceae Correct Choice

D. Compositae

E. Urticaceae
The Apiaceae family (formerly Umbelliferae) includes parsley, celery, parsnip, hogweed, and fennel,
all potential causes of phytophotodermatitis


42) When bitten with the spider in the photograph, the active agent in the venom is:

A. Sphingomyelinase-D

B. Alpha lactrotoxin Correct Choice

C. Lipase

D. Hyaluronidase

E. Histamine
This is a black widow spider which can be identifiec by its shiny black body
with characteristic white hourglass marking on the abdomen


43) The vector for Babesia microti is also a vector for:

A. Rickettsia rickettsii

B. None of these answers are correct

C. Francisella tularensis




                                                  11
D. Borrelia burgdorferiCorrect Choice

E. All of these answers are correct
The vector for Babesia microti, the pathogenic organism of babesiosis, is Ixodes dammini, which is
also a vector for Borrelia burgdorferi, the pathogenic organism of Lyme borreliosis. Francisella
tularensis (the pathogenic organism of tularemia) is transmitted by Dermacentor andersoni and
Dermacentor variabilis. Rickettsia rickettsii (the pathogenic organism of Rocky Mountain Spotted
Fever) is transmitted by Dermacentor andersoni, Dermacentor variabilis, and Amblyomma
americanum


44) Allergic contact dermatitis is a:

A. Type III hypersensitivity

B. Type II hypersensitivity

C. Type I hypersensitivity

D. Undetermined hypersensitivity

E. Type IV hypersensitivity Correct Choice
NEEDS EXPLANATIONS


45) Match the photo to the plant identified:

A. Mango leaves

B. Poison ivy

C. Gingko

D. Poison oak Correct Choice

E. Poison sumac
Poison oak is identified by leaf shape - irregular lobing and varied leaf size


46) The insect pictured is:

A. A vector of relapsing fever

B. The cause of maculae cerulea

C. A vector of Lyme borreliosisCorrect Choice

D. Ornithonyssus

E. A vector of babesiosis
The insect pictured is a female Amblyomma americanum tick. This tick can serve as a vector for
Lyme borreliosis and Rocky Mountain Spotted Fever. Relapsing fever is transmitted by Ornithodoros
moubata. Babesiosis is trasnmitted by Ixodes dammini. Maculae cerulea are seen in severe cases of
crab lice. Ornithonyssus is the fowl mite


47) Which of the following is the most common cause of dermatitis in florists?

A. Sesquiterpene lactones

B. Calcium oxalate Correct Choice

C. Primin




                                                   12
D. Tuliposide A

E. Mechanical dermatitis
Calcium oxalate is present in the bulbs and stems of Narcissus (daffodil


48) Snake bite antivenin is derived from:

A. Pooled IVIg

B. Horse antibodyCorrect Choice

C. Human antibody

D. Mouse antibody

E. Antigen extracted directly from the attacking snake
Following envenomation, antivenins are derived from horses


49) All of the following plants commonly cause a phytophotodermatitis except:

A. Celery

B. Ficus carica

C. Hawaiian lei flowers

D. Myroxylon balsamumCorrect Choice

E. All of these plants commonly cause a phytophotodermatitis
Myroxylon balsamum is the source of Balsam of Peru, a common sensitizer of allergic contact
dermatitis. Hawaiian lei flowers, Ficus carica (fig tree), and celery all are known to commonly cause
phytophotodermatitis


50) On slide preparation, many organisms were noted on this patient, as well as occasional blue
macules on the buttocks. He most likely has:


A. Chigoe fleas

B. Pediculus capitis infestation

C. Pthirus pubis infestationCorrect Choice

D. Sarcoptes scabiei infestation

E. Pediculus humanus infestation
It is often very easy to find crab lice on physical exam in contrast with scabies. Head lice
infestations are usually limited to the scalp. Body lice are not often found on the patient, more
commonly in clothing seams. Maculae caerulea can be seen in widespread involvement as in this
case


51) Which of the following plants are members of the family Rutaceae?

A. Burning bush

B. Rue

C. All of these answers are correctCorrect Choice

D. None of these answers are correct




                                                    13
E. Bergamot orange
The family Rutaceae includes lime, rue, burning bush, bergamot orange, and Hawaiian lei flowers.
Plants from this family commonly cause phytophotodermatitis


52) The toxin holothurin, released by which of the following marine creatures, can cause
conjunctivitis in exposed swimmers?


A. Sea cucumberCorrect Choice

B. None of these answers are correct

C. Chironex fleckeri

D. Edwardsiella lineate

E. Linuche unguiculata
Sea cucumbers are cucumber-shaped bottom-dwellers that can cause conjunctivitis due to release
of a toxin called holothurin to which nearby swimmers can be exposed. The larva of Linuche
unguiculata (thimble jelly fish) and Edwardsiella lineate (sea anemone) are causes of seabather's
eruption (or sea lice). Chironex fleckeri is the box jellyfish; its stings are extremely toxic and may
lead to shock and death, especially in small adults and children


53) Cantharidin creates a vesicle by which of the following mechanisms?

A. Subcorneal layer separation

B. Instability of the hemidesmosome

C. Antibodies to desmosomal proteins

D. Local recruitment of neutrophils

E. Disruption of keratinocyte-keratinocyte adhesion Correct Choice
Desmosomal adhesions are interrupted by proteases with eventual tonofilament detachment


54) Patients with an allergy to the compositae family may have an allergic reaction to which of the
following drugs?


A. Pyruvic acid

B. Pyrethrins Correct Choice

C. Psoralens

D. Propylene glycol

E. Pseudoephedrine
Pyrethrins are derived from chrysanthemum flowers. Patients with allergy to these flowers or
ragweed may experience allergic symptoms after using pyrethrins


55) This patient complains of intense itching at this site. KOH examination is negative. She denies
any recent travel, however does take her 2 year old to the local playground where there is a
sandbox. Which of the following is appropriate?


A. Give her clobetasol ointment

B. Slowly extract the organism

C. Inform the patient she is contagious and should isolate herself from her children




                                                  14
D. Treat with po ivermectin Correct Choice

E. Obviously this is dermatophyte despite a negative KOH. Treat with topical antifungals
This is hookworm, or cutaneous larva migrans; systemic ivermectin or topical thiobendazole would
be appropriate treatments. The causative organism is the larvae of Ancylostoma braziliensis or A.
canium


56) This creature can do the following EXCEPT:

A. Cause a painful sting

B. Transmit Yersinia pestisCorrect Choice

C. Cause anaphylaxis

D. Be used as a natural control for the tick population

E. Build large characteristic mounds
This is a fire ant. In some parts of the world, they are used as a natural tick control as they are
scavengers and consumers of tick eggs. They can cause anaphylaxis in an allergic person. They do
not transmit disease. The mounds are characteristic in appearance


57) Babesia microti:

A. Is transmitted by Ixodes dammini

B. All of these answers are correctCorrect Choice

C. None of these answers are correct

D. Is an intracellular parasite

E. Is pathogenic in an infection that is endemic on Martha's Vineyard
Babesiosis is caused by an intracellular RBC parasite, Babesia microti. It is transmitted by the larvae
of Ixodes dammini. This disease is endemic in eastern Long Island, Martha's Vineyard, and
Nantucket, and carries an increased risk in those with T-cell depression or after splenectomy. The
condition is associated with fever, drenching sweats, myalgias, and hemolytic anemia


58) Which chemical is present in large quantities of this insect's venom?

A. Lipase

B. Hyaluronidase

C. Sphingomyelinase D

D. Alpha lactrotoxin

E. HistamineCorrect Choice
The spider shown is the Wolf spider (Lycosidae). This Australian spider's bites contains large
amounts of histamine. Bites are painful with edema, erythema and subsequent lymphangitis


59) Which of the following is false?

A. Divers are at risk of contacting these creatures

B. Only pigmented varieties pose any riskCorrect Choice

C. These spines may contain toxins




                                                    15
D. This is a sea urchin

E. Granulomas can form around the spines if not removed
This is a sea cucumber. A small fraction of spines contain toxins which affect humans. Divers may
step on these creatures which is quite painful. Spines should be removed to avoid foreign body
reactions


60) Which of the following mites is also known as the chigger and often causes intense pruritus on
the ankles, legs, or belt line?


A. Glyciphagus

B. Allodermanyssus sanguineus

C. Trombicula alfreddugesi Correct Choice

D. Demodex

E. Ornithonyssus
Trombicula alfreddugesi (chigger mite, harvest mites) frequently causes intense pruritus on the
ankles, legs, or belt line. Contact occurs during summer and fall. In sensitized individuals, papular
urticaria, vesiculation, or a granulomatous reaction with fever and lymphadenopathy may occur.
Allodermanyssus sanguineus (house mouse mite) is the vector of rickettsial pox. Demodex
folliculorum is a mite that can live within the sebaceous glands of hair follicles and may be
important in some subsets of patients with acne rosacea, as well as patients with HIV infection and
folliculitis. Glyciphagus (cheese mite) is the cause of grocer's itch. Ornithonyssus (fowl mite) can
carry Western equine encephalitis


61) Contact urticaria is a:

A. Type II hypersensitivity

B. Type I hypersensitivity Correct Choice

C. Type III hypersensitivity

D. Undetermined hypersensitivity

E. Type IV hypersensitivity
NEEDS EXPLANATIONS


62) Which of these spiders is bright green with red spots, with black spines on its legs?

A. Phidippus formosus

B. Loxosceles reclusa

C. Latrodectus mactans

D. Peucetia viridansCorrect Choice

E. Centruroides sculpturatus
Peucetia viridans (green lynx spider) is common in the southern United States. It is bright green
with red spots; the legs have black spines. The bite is painful, with tenderness and pruritus.
Phidippus formosus (jumping spider) is an aggressive spider with dark body hairs and various white
patterns. They are the most common biting spiders in the United States. Loxosceles reclusa (brown
recluse) is a tan spider with a violin-shaped marking on the abdomen. The bite may be painless,
however the patient may develop erythema, a vesicle, and eventual necrosis. Latrodectus mactans
(black widow) is a large, black, shiny spider with an hourglass shaped white marking on the




                                                 16
abdomen. Bites are acutely painful, and may swell but do not cause necrosis; the venom contains
neurotoxins. Centruroides sculpturatus is a primary scorpion species in the U.S


63) Mechanical irritant dermatitis can be caused by:

A. GlochidsCorrect Choice

B. Bromelin

C. Stinging nettles

D. Capsacin

E. All of these answers are correct
Mechanical irritant dermatitis can be caused by the direct effects of thorns and barbs. Small glochids
(barbed hairs or brisles) or large thorns (on cacti for example) can become imbedded into the skin
causing injury and resultant dermatitis. Secondary infection can be caused by microorganism
inoculation


64) Which of the statements regarding Tegenaria agrestis is correct?

A. This creature is the leading cause of necrotic arachnidism in several states of the Pacific
NorthwestCorrect Choice

B. None of these answers are correct

C. Contact with this creature results in erythematous papules, hemorrhage, or purpura in a classic
"tram-track" pattern

D. Stings from this scorpion can produce cardiovascular complications

E. This ant injects a venom that contains a hemolytic factor, solenopsin D
Tegenaria agrestis (the Hobo spider) is the leading cause of necrotic arachnidism in several states of
the Pacific Northwest. These spiders are large, with a herringbone-striped pattern on the abdomen.
Contact with caterpillars (Lepidoptera) may produce lesions in a classic "tram-track" pattern.
Solenopsis (fire ants) ants inject a venom that contains a hemolytic factor, solenopsin D, that
causes a release of histamine and other vasoactive amines from mast cells


65) Insects from the species pictured:

A. Trasmit yellow fever

B. Transmit malaria and yellow fever

C. Transmit dengue fever

D. Trasmit yellow fever and dengue feverCorrect Choice

E. Transmit malaria
The insect pictured is an Aedes mosquito. Aedes transmits yellow fever and dengue fever.
Anopheles mosquitoes transmit malaria


66) A 52 year-old gardener presents with innumerable linear hyperpigmented streaks across the
chest and arms. He states that several weeks previously he had a painful, erythematous, blistering
eruption at the same site. The most likely botanical cause for this eruption is:


A. Plants of the Alliaceae family

B. Lime




                                                   17
C. HogweedCorrect Choice

D. Peruvian lily

E. Ragweed
The gardener has so-called "strimmer dermatitis." The use of a weed-whacker may result in a spray
of weeds (such as cow parsley, wild chervil, and hogweed) leading to this phytophotodermatitis on
exposed areas such as the chest and arms


67) Identify this plant:

A. Alstromeria

B. Toxicodentron

C. Myroxylon balsamum

D. PrimroseCorrect Choice

E. Ragweed
This is primrose (primulaceae) a cause of allergic contact dermatitis


68) Necrosis may be seen in which of the following:

A. Allergic contact dermatitis

B. Irritant contact dermatitis Correct Choice

C. Dermatographism

D. Contact urticaria

E. Acute urticaria
Necrosis may be seen in severe irritant reactions. The other reactions do not involve necrosis


69) Which creature is a human flea that can also be seen on dogs?

A. Liponyssoides sanuineus

B. Pulex irritansCorrect Choice

C. Lytta vesicatoria

D. Ctenocephalides canis

E. Pediculus humanus
Pulex irritans is the human flea; it can also be seen on dogs. More frequent infestations on domestic
animals are by Ctenocephalides felis and canis (on cats and dogs respectively). Lytta vesicatoria is a
blister beetle from which cantharadin is derived. Pediculus humanus var. corporis is the human body
louse.


70) Which of the following is a complication of Thermage treatment?

A. Hyperpigmentation

B. LipoatrophyCorrect Choice

C. Keloid formation

D. Granuloma formation




                                                 18
E. Herpes simplex infection
Thermage (ThermaCool) is a nonablative radiofrequency-based system which uses volumetric
heating to induce tightening of the skin and dermal remodeling. Lipoatrophy is a late-occurring
potential complication of this technology


71) Which of the following is not a hallmark of allergic contact dermatitis?

A. Eosinophils

B. Neutrophils Correct Choice

C. Langerhans cells

D. CD4+ Lymphocytes

E. Vesicles
Langerhans cells present the antigens to CD4+ lymphocytes in the dermis and after traveling
through lymphatics to regional lymph nodes. Eosinophils may also be present. Neutrophils are not
characteristic of this reaction


72) The female Sarcoptes scabiei var. hominis mite can lay up to 100 eggs. It has an ovoid, flat
body and how many short legs?


A. 10

B. 6

C. None of these answers are correct

D. 12

E. 8Correct Choice
The female Sarcoptes scabiei var. hominis mite has an ovoid, flat body and 8 short legs


73) Which snake is part of the Crotalidae family?

A. Garden snake

B. Coral snake

C. RattlesnakeCorrect Choice

D. Cobra

E. Water snake
The Crotalidae family includes rattlesnakes and copperheads. The coral snake is part of the elapidae
family


74) A 22 year-old man presents with an irregular laceration on the dorsal metacarpals following a
fist fight in which he struck someone in the mouth. The tissue surrounding the wound is
erythematous, edematous, indurated, warm, and tender. There is some purulence of the wound.
You consider infection with:


A. Erysipelothrix rhusiopathiae

B. Eikenella corrodensCorrect Choice

C. Pasteurella multocida




                                                  19
D. Pasteurella canis

E. Capnocytophaga
Human bites may result in infection with Eikenella corrodens, a gram-negative bacillus that is part
of the normal flora of the human mouth. This infection can follow fist fights with blows to the mouth


75) Which of the following infections is commonly known to occur from contact with infected sheep?

A. Oculoglandular syndrome of Parinaud

B. Infectious pustular dermatitis

C. Cutaneous anthrax

D. Infectious pustular dermatitis and cutaneous anthraxCorrect Choice

E. Erysipeloid
Infectious pustular dermatitis is another name for orf, or ecthyma contagiosum. It is transmitted to
humans by a parapoxvirus in sheep, goats, and reindeer. Cutaneous anthrax is also called
"woolsorter's disease" and is caused by Bacillus anthracis, a gram-positive rod transmitted from
contact with carcasses of infected sheep, cows, goats, and horses. The oculoglandular syndrome of
Parinaud is the combination of granulomatous conjunctivitis and preauricular lymphadenopathy seen
in cat-scratch disease


76) A goat farmer presents with fever and flu-like symptoms, as well as violaceous papulonodules
on the trunk and lower extremities. You consider Malta fever in your differential diagnosis. Which of
the following statements regarding this diagnosis is correct?


A. Skin lesions are infrequently seen

B. None of these answers are correct

C. All of these answers are correctCorrect Choice

D. Treatment is with rifampin

E. Treatment is with doxycycline
Malta fever is another name for Brucellosis. This infection is caused by ingesting raw goat milk and
unpasteurized goat cheese infected with Brucellae (gram-negative rods). It is infrequent in the US,
occurring mainly in veterinarians and farmers. Acute brucellosis has very non-specific features and
presents as a flu-like illness. Skin lesions are infrequent (20% of cases) but are usually violaceous
papulonodules on the trunk and lower extremities. Treatment is with doxycycline or rifampin


77) Which disease is transmitted by this insect?

A. Visceral leishmaniasis

B. African sleeping sickness

C. OnchocerciasisCorrect Choice

D. Chagas disease

E. Dracunculiasis
The insect depicted is the black fly, Simulium species. The black fly is the vector for onchocerciasis,
which is caused by onchocerca volvulus. Clinical manifestation of this disease include atrophy,
depigmentation and thickening of the skin. In addition, the infection may cause "river blindess" and
onchocercomas, which are nodules containing microfilariae


78) This organism is:


                                                    20
A. Chigoe flea

B. Pthirus pubis Correct Choice

C. Pediculus capitis

D. Pulex irritans

E. Pediculus humanus
This is a crab louse. It is shorter and wider than the head and body louse. Note the crab-like claws


79) A 54 year-old man with psoriatic arthritis on methotrexate and infliximab calls you after he is
bitten deeply on the ankle by his daughter's 2 year-old Chihuahua. He is concerned that his
immunocompromised status puts him at risk for infection and potential complications from this bite.
You correctly tell him:


A. Capnocytophaga from dog bites can cause sepsis in immunocompromised victimsCorrect Choice

B. As long as the dog has been vaccinated, he need not worry about serious infections

C. Eikenella corrodens is the most common cause of infection following dog bites

D. Pasteurella multocida infection is frequent after dog bites but not cat bites

E. Rickettsial pox can be seen after dog bites
Local infection and cellulitis is a concern following domesticated animal bites; vaccination may
prevent against rabies, but not other infections. Cat bites most frequently result in Pasteurella
multocida infections, while dog bites result in Pasteurella canis. However, most bites are
polymicrobial with mixed aerobes and anaerobes. Infrequently systemic infections may result in
brain or lung abscess, endocarditis, or sepsis. Sepsis is more frequent in immunocompromised
victims and may be due to Capnocytophaga or Pasteurella


80) What is the initial treatment of choice for this dermatitis?

A. Warm compresses

B. Topical steroid

C. Topical antibiotics

D. 5% acetic acidCorrect Choice

E. Observation
The linear appearance of the dermatitis is typical of jellyfish stings. To prevent further nematocyte
activation, 5% acetic acids should be applied for 15-30 minutes. Papain found in meat tenderizers
or 70% isopropyl alcohol may also be applied. Warm compresses are contraindicated as they may
increase systemic absorption of the venom


81) Pineapples contain which of the following proteolytic enzymes?

A. None of these answers are correct

B. BromelinCorrect Choice

C. Phorbol esters

D. Ranunculin

E. All of these answers are correct




                                                   21
Bromelin is a proteolytic enzyme found in pineapples. It is granted access to dermal vessels by
calcium oxalate-induced fissures. Phorbol esters are found in plants of the Euphorbiaceae family.
Ranunculin is an irritant found in buttercups (Ranunculaceae family


82) Which of the following infections is caused by a gram-positive rod?

A. Brucellosis

B. Erysipeloid of RosenbachCorrect Choice

C. Cat-scratch disease

D. Milker's nodule

E. Glanders
Erysipeloid of Rosenbach is an infection with Erysipelothrix insidiosa or Erysipelothrix rhusiopathiae,
gram-positive rods. Brucellosis is caused by infection with Brucellae (gram-negative rod). Cat-
scratch disease is caused by infection with Bartonella henselae, a gram-negative rod. Glanders is
caused by infection with Pseudomonas mallei, a gram-negative rod. Milker's nodule is caused by the
paravaccinia viurs, a parapoxvirus related to the orf virus


83) Which of the following vectors transmits Borrelia duttonii?

A. Rhipicephalus sanguineus

B. Amblyomma americanum

C. Ornithodoros moubataCorrect Choice

D. Ixodes pacificus

E. none of the above
Borrelia duttonii is a pathogenic organism of relapsing fever. It is transmitted by Ornithodoros
moubata. Ixodes pacificus is one of the vectors of Lyme Borreliosis. Rhipicephalus sanguineus is a
vector of Ehrlichiosis. Amblyomma americanum is a vector of Lyme Borreliosis and Rocky Mountain
Spotted Fever


84) Ehrlichiosis is classified in two primary forms. These are:

A. Intracellular and extracellular

B. Primary and secondary

C. None of these answers are correct

D. Endemic and epidemic

E. Monocytic and granulocyticCorrect Choice
Ehrlichiosis comes in two forms: monocytic, which invades mononuclear WBCs, and granulocytic, in
which peripheral PMNs show intracytoplasmic inclusions. Patients present with fever, chills,
headache, and myalgias, along with leukopenia, anemia, and thrombocytopenia


85) Capsaicin causes which of the following?

A. Blocking the influx of sodium ions into the cell

B. Decreased firing of nerve fibers transmitting pain

C. Increased substance P release Correct Choice




                                                      22
D. Creating a barrier between nerves and their stimuli

E. Decreased substance P release
Repeated substance P release leads to eventual depletion of this agent which causes pain
transmission


86) Which of the following infections is caused by a gram-negative rod?

A. Bacillary angiomatosis

B. Malta fever

C. None of these answers are correct

D. Farcy

E. All of these answers are correctCorrect Choice
Malta fever is another name for Brucellosis, an infection caused by Brucellae (gram-negative rod).
Farcy is another name for Glanders, an infection with the gram-negative rod Pseudomonas mallei.
Bacillary angiomatosis is caused by infection with Bartonella henselae, a gram-negative rod


87) Sesquiterpene lactones are found in which of the following plants?

A. Alstromeria

B. Pinus palustris

C. Gingko

D. ArtichokeCorrect Choice

E. Chive
Sesquiterpene lactones are sensitizers found in plants of the Asteraceae (Compositae) family.
Members include ragweed, pyrethrum, chrysanthemum, weeds, feverfew, and artichoke. The
sensitzer found in alstromeria (Peruvian lily) is tuliposide A (a glycoside). The Pinus palustris tree is
the source of colophony. Chives contain diallyl disulfide. Gingko fruit pulp is a sensitizer


88) Match the photo to the plant identified:

A. Compositae

B. Alstromeria Correct Choice

C. Primulaceae

D. Anacardiacea

E. Alliaceae
Alstromeria is pictured


89) Which of the following contains active furocoumarins?

A. Fig Correct Choice

B. Lichens

C. Mango

D. Chrysanthemum




                                                    23
E. Primrose
The fig tree may cause a phytophotodermatitis


90) The necrosis and hemolysis that can result from a brown recluse spider bite may be attributed
to which of the following toxins?


A. Lipase

B. Sphingomyelinase-DCorrect Choice

C. Alpha lactrotoxin

D. Hyaluronidase

E. Histamine
The brown recluse spider (Loxosceles reclusa) is a tan spider with a violin-shaped marking on the
abdomen. Its bite may be painless, however the patient may develop erythema, a vesicle, and
eventual necrosis. This may result in the "red, white, and blue sign." Systemic reactions
(viscerocutaneous loxoscelism) may occur. The venom contains several toxins, however
sphingomyelinase D seems to be causative of the necrosis and hemolysis


91) This patient acquired an infection with a microorganism that may have been inoculated into the
skin from contact with:


A. Grasses

B. Rose thorns

C. All of these answers are correctCorrect Choice

D. Sphagnum moss

E. None of these answers are correct
This patient has sporotrichosis, which is caused by Sporothrix schenckii. Sporothrix schenckii can be
inoculated into the skin by grasses, sphagnum moss, and rose thorns


92) If suspicious for scabies infestation, which of the following areas is least likely to be involved?

A. External genitalia

B. Umbilicus

C. Fingerwebs

D. Nipples

E. Occipital scalpCorrect Choice
Erythematous papules and burrows are commonly seen in webspaces, nipples, external genitalia,
umbilicus and axilla in sarcoptes scabiei infestation


93) Which of the following statements regarding orf is correct?

A. Orf is also known as "pseudocowpox."

B. Lesions last indefinitely without treatment

C. The papillomatous stage of lesion progression preceeds the regenerative stage

D. The virus is very sturdy and survives many months in cold weatherCorrect Choice




                                                    24
E. The papular stage of lesion progression follows the target stage
Orf, also known as ecthyma contagiosum or infectious pustular dermatitis, is transmitted to humans
by a parapoxvirus in sheep, goats, and reindeer. This is a self-limited condition with an excellent
prognosis lasting approximately 6 weeks. Lesion morphology changes through a series of stages:
papular (red, elevated) -> target (nodule with red center, white middle ring, red periphery) -->
acute (red, weeping) --> regenerative (thin, dry crust overlying lesion) --> papillomatous -->
regressive. The virus is very sturdy and survives many months in cold weather. Milker's nodule,
caused by the paravaccinia virus (a parapoxvirus, related to the orf virus), infects cows and can be
transmitted to dairy workers, and is also known as pseudocowpox


94) The majority of phytophotodermatitis is caused by plants from which family?

A. ApiaceaeCorrect Choice

B. Compositae

C. Liliaceae

D. Alliaceae

E. None of these answers are correct
The Apiaceae family (formerly Umbelliferae) includes parsley, celery, parsnip, hogweed, and fennel,
and causes the majority of phytophotodermatitis


95) Thorns and barbs on blackberry bushes have been known to cause secondary infection through
inoculation of:


A. None of these answers are correct

B. Mycobacterium ulcerans

C. Sporothrix schenckii

D. All of these answers are correct

E. Mycobacterium kansaiiCorrect Choice
Thorns and barbs on blackberry bushes have been known to cause secondary infection through
inoculation of Mycobacterium kansaii. Sporothrix schenckii can be innoculated by grasses,
sphagnum moss, and rose thorns. Mycobacterium ulcerans has been innoculated by spiky tropical
vegetation


96) Match the photo to the plant identified:

A. Toxicodendron

B. Compositae Correct Choice

C. Primulaceae

D. Alstromeria

E. Alliaceae
This is ragweed, a member of the compositae (or asteraceae) family


97) Plants from the family Moraceae include which of the following?

A. All of these answers are correct

B. Fennel



                                                 25
C. None of these answers are correct

D. Burning bush

E. Ficus caricaCorrect Choice
Ficus carica (fig tree) is a member of the family Moraceae and a cause of phytophotodermatitis.
Burning bush is a member of the family Rutaceae. Fennel is a member of the family Apiaceae


98) Identify this tick:

A. Dermacentor andersoni

B. Rhipicephalus sanguineus

C. Dermacentor variabilis

D. Amblyomma americanumCorrect Choice

E. Ixodes dammini
This is Amblyomma americanum which transmits Lyme as well as RMSF


99) Which of the following is a common cause of seabather's eruption?

A. Linuche unguiculata

B. Thimble jelly fish

C. All of these answers are correctCorrect Choice

D. Edwardsiella lineate

E. Sea anemone
Seabather's eruption (or sea lice) can be sporadic or part of an outbreak. Clinically, it appears as
dermatitis beneath areas covered by swim suits. The geographic distribution is most frequent from
the Caribbean to Bermuda, however there have been three outbreaks at Long Island, NY beaches. A
variety of cnidarian larvae can cause this eruption, including thimble jelly fish (Linuche unguiculata)
and sea anemone (Edwardsiella lineate). Quick removal of bathing suits and rinsing can help with
prevention. Symptom relief is the mainstay of therapy


100) Phytophotodermatitis caused by skin exposure to limes is caused by:

A. D-limonene

B. Furocoumarin Correct Choice

C. Tuliposide A

D. Sesquiterpene lactone

E. Ascorbic acid
Phytophotodermatatitis (phototoxic eruptions from plants) are relatively commonly seen in florists,
nursery personnel, gardeners, and farm workers. Plants that cause this eruption may produce
photoxins such as furocoumarins Fig, fennel, lime, lemon, parsley, celery, dill, and hogweed are all
causes of phytophotodermatitis


101) “Strimmer” dermatitis is caused by which of the following?

A. M. marinum




                                                    26
B. Lime

C. Pseudomonas

D. Cryptococcus

E. Chervil Correct Choice
Strimmer dermatitis is a phytophotodermatitis caused by spray of plant matter when using a weed-
whacker or similar trimming tool


102) Identify this insect:

A. Water flea

B. Tsetse fly

C. Mango fly

D. Reduviid bugCorrect Choice

E. Phlebotomous sandfly
This a picture of the reduviid bug, also called the kissing bug or assassin bug. It is the vector for
American Trypanosomiasis (Chagas disease), which is caused by Trypanosoma cruzii. The reduviid
bug prefers to bite at mucocutaneous junctions


103) Solenopsin D is a:

A. Phospholipase

B. Piperdine derivative

C. Hemolytic factor

D. Hemolytic factor, piperdine derivative, histamine-releaserCorrect Choice

E. Histamine-releaser
Solenopsin D is a venom which is a hemolytic factor, a piperidine derative and leads to histamine
release. The venom from honeybeeds contains phospholipase A


104) The majority of naturally-occurring cases of anthrax:

A. Are extra-cutaneous

B. Are acquired through ingestion of spores

C. Are pulmonary

D. Are cutaneousCorrect Choice

E. Are oropharyngeal
The majority (95%) of naturally-occurring cases of anthrax are of the cutaneous form, acquired
from direct contact with the carcasses of dead sheep, cows, goats, and horses. Naturally-occurring
pulmonary, gastrointestinal (acquired by ingestion of spores) and oropharyngeal infection with
anthrax is less common


105) Ticks of the species shown:

A. Can transit tularemia but not Rocky Mountain Spotted Fever

B. Can transmit Rocky Mountain Spotted Fever and tularemiaCorrect Choice



                                                  27
C. Can transmit Lyme borreliosis but not Rocky Mountain Spotted Fever or tularemia

D. Can transmit Rocky Mountain Spotted Fever and Lyme borreliosis

E. Can transmit Lyme borreliosis and tularemia
The tick pictured is a male Dermacentor variabilis tick. Dermacentor variabilis can transmit Rocky
Mountain Spotted Fever and tularemia. Lyme borreliosis is transmitted by Ixodes dammini, Ixodes
pacificus, and Amblyomma americanum.




                                                 28

Weitere ähnliche Inhalte

Was ist angesagt?

ETAS_MCQ_10 manifestations of systemic diseases1
ETAS_MCQ_10 manifestations of systemic diseases1ETAS_MCQ_10 manifestations of systemic diseases1
ETAS_MCQ_10 manifestations of systemic diseases1Derma202
 
ETAS_MCQ_03 b genodermatoses
ETAS_MCQ_03 b genodermatosesETAS_MCQ_03 b genodermatoses
ETAS_MCQ_03 b genodermatosesDerma202
 
ETAS_MCQ_05 dermatopathology
ETAS_MCQ_05 dermatopathologyETAS_MCQ_05 dermatopathology
ETAS_MCQ_05 dermatopathologyDerma202
 
ETAS_MCQ_02 immunodermatology
ETAS_MCQ_02 immunodermatologyETAS_MCQ_02 immunodermatology
ETAS_MCQ_02 immunodermatologyDerma202
 
Previous year question on lichen planus based on neet pg, usmle, plab and fmg...
Previous year question on lichen planus based on neet pg, usmle, plab and fmg...Previous year question on lichen planus based on neet pg, usmle, plab and fmg...
Previous year question on lichen planus based on neet pg, usmle, plab and fmg...Abhishek Gupta
 
Aspergillus and systemic mycoses
Aspergillus and systemic mycosesAspergillus and systemic mycoses
Aspergillus and systemic mycosesR Lin
 
Ocular toxoplasmosis
Ocular toxoplasmosisOcular toxoplasmosis
Ocular toxoplasmosisBipin Bista
 
Ocular parasitic infection
Ocular parasitic infectionOcular parasitic infection
Ocular parasitic infection9925752690
 
CLS Histoplasmosis.pptx
CLS Histoplasmosis.pptxCLS Histoplasmosis.pptx
CLS Histoplasmosis.pptxYadav Raj
 
Yeasts of medical importance - Candida and Cryptococcus
Yeasts of medical importance - Candida and CryptococcusYeasts of medical importance - Candida and Cryptococcus
Yeasts of medical importance - Candida and CryptococcusAnuswedha Ananthan
 
CLS Blastomyces dermatitidis.pptx
CLS Blastomyces dermatitidis.pptxCLS Blastomyces dermatitidis.pptx
CLS Blastomyces dermatitidis.pptxYadav Raj
 
CLS Mycetoma.pptx
CLS Mycetoma.pptxCLS Mycetoma.pptx
CLS Mycetoma.pptxYadav Raj
 
subcutaneous Mycosis & systemic mycosis
subcutaneous Mycosis & systemic mycosis subcutaneous Mycosis & systemic mycosis
subcutaneous Mycosis & systemic mycosis ReiyaBosco
 
CLS Coccidioidomyces.pptx
CLS Coccidioidomyces.pptxCLS Coccidioidomyces.pptx
CLS Coccidioidomyces.pptxYadav Raj
 

Was ist angesagt? (20)

ETAS_MCQ_10 manifestations of systemic diseases1
ETAS_MCQ_10 manifestations of systemic diseases1ETAS_MCQ_10 manifestations of systemic diseases1
ETAS_MCQ_10 manifestations of systemic diseases1
 
ETAS_MCQ_03 b genodermatoses
ETAS_MCQ_03 b genodermatosesETAS_MCQ_03 b genodermatoses
ETAS_MCQ_03 b genodermatoses
 
ETAS_MCQ_05 dermatopathology
ETAS_MCQ_05 dermatopathologyETAS_MCQ_05 dermatopathology
ETAS_MCQ_05 dermatopathology
 
ETAS_MCQ_02 immunodermatology
ETAS_MCQ_02 immunodermatologyETAS_MCQ_02 immunodermatology
ETAS_MCQ_02 immunodermatology
 
Previous year question on lichen planus based on neet pg, usmle, plab and fmg...
Previous year question on lichen planus based on neet pg, usmle, plab and fmg...Previous year question on lichen planus based on neet pg, usmle, plab and fmg...
Previous year question on lichen planus based on neet pg, usmle, plab and fmg...
 
Aspergillus and systemic mycoses
Aspergillus and systemic mycosesAspergillus and systemic mycoses
Aspergillus and systemic mycoses
 
N059 06 17-2011
N059 06 17-2011N059 06 17-2011
N059 06 17-2011
 
Mrcp Part 2 Witten Exam
Mrcp Part 2 Witten ExamMrcp Part 2 Witten Exam
Mrcp Part 2 Witten Exam
 
Ophthalmic parasitology
Ophthalmic parasitologyOphthalmic parasitology
Ophthalmic parasitology
 
Ocular toxoplasmosis
Ocular toxoplasmosisOcular toxoplasmosis
Ocular toxoplasmosis
 
Ocular parasitic infection
Ocular parasitic infectionOcular parasitic infection
Ocular parasitic infection
 
CLS Histoplasmosis.pptx
CLS Histoplasmosis.pptxCLS Histoplasmosis.pptx
CLS Histoplasmosis.pptx
 
Yeasts of medical importance - Candida and Cryptococcus
Yeasts of medical importance - Candida and CryptococcusYeasts of medical importance - Candida and Cryptococcus
Yeasts of medical importance - Candida and Cryptococcus
 
Herpes virus
Herpes virusHerpes virus
Herpes virus
 
CLS Blastomyces dermatitidis.pptx
CLS Blastomyces dermatitidis.pptxCLS Blastomyces dermatitidis.pptx
CLS Blastomyces dermatitidis.pptx
 
CLS Mycetoma.pptx
CLS Mycetoma.pptxCLS Mycetoma.pptx
CLS Mycetoma.pptx
 
subcutaneous Mycosis & systemic mycosis
subcutaneous Mycosis & systemic mycosis subcutaneous Mycosis & systemic mycosis
subcutaneous Mycosis & systemic mycosis
 
CLS Coccidioidomyces.pptx
CLS Coccidioidomyces.pptxCLS Coccidioidomyces.pptx
CLS Coccidioidomyces.pptx
 
Leprosy
LeprosyLeprosy
Leprosy
 
Systemic mycosis
Systemic mycosisSystemic mycosis
Systemic mycosis
 

Ähnlich wie ETAS_MCQ_14 plants and creatures of dermatologic significance

Medical entomology lecture ppt
Medical entomology lecture pptMedical entomology lecture ppt
Medical entomology lecture pptDr.Farhana Yasmin
 
ENTO 301 – MEDICAL & VETERINARY ENTOMOLOGY (Lectures 1-5) ENTO 301 – MEDICA...
ENTO 301 – MEDICAL & VETERINARY ENTOMOLOGY (Lectures 1-5) 	 ENTO 301 – MEDICA...ENTO 301 – MEDICAL & VETERINARY ENTOMOLOGY (Lectures 1-5) 	 ENTO 301 – MEDICA...
ENTO 301 – MEDICAL & VETERINARY ENTOMOLOGY (Lectures 1-5) ENTO 301 – MEDICA...MedicineAndHealth
 
Medical entamology
Medical entamologyMedical entamology
Medical entamologySuvarna Wagh
 
Entomopathogenenic nematodes
Entomopathogenenic nematodesEntomopathogenenic nematodes
Entomopathogenenic nematodesShariqaJan
 
Helminths in adult population in countries.pptx
Helminths in adult population in countries.pptxHelminths in adult population in countries.pptx
Helminths in adult population in countries.pptxbarakaakute
 
Human resources section_8-textbook_on_public_health_and_community_medicine
Human resources section_8-textbook_on_public_health_and_community_medicineHuman resources section_8-textbook_on_public_health_and_community_medicine
Human resources section_8-textbook_on_public_health_and_community_medicinePrabir Chatterjee
 
Tick Borne Diseases of Public Significance and Integrated Vector Management
Tick Borne Diseases of Public Significance and Integrated Vector ManagementTick Borne Diseases of Public Significance and Integrated Vector Management
Tick Borne Diseases of Public Significance and Integrated Vector ManagementDr Shifa Ul Haq
 
Flies and mosquito related medical conditions and vector control ppt
Flies and mosquito related medical conditions and vector control pptFlies and mosquito related medical conditions and vector control ppt
Flies and mosquito related medical conditions and vector control pptaaminaabokor09
 
Tick borne parasitic infections
Tick borne parasitic infectionsTick borne parasitic infections
Tick borne parasitic infectionsUwamose MNO
 
2010.05.17_FRACP_Insect_Sting_Allergy-2.pdf
2010.05.17_FRACP_Insect_Sting_Allergy-2.pdf2010.05.17_FRACP_Insect_Sting_Allergy-2.pdf
2010.05.17_FRACP_Insect_Sting_Allergy-2.pdfAssemAssem6
 
Rickettsia & chlamydia presentation.
Rickettsia & chlamydia presentation.Rickettsia & chlamydia presentation.
Rickettsia & chlamydia presentation.Bruno Mmassy
 
Citrobacter frendii infections in Reptiles
Citrobacter frendii infections in ReptilesCitrobacter frendii infections in Reptiles
Citrobacter frendii infections in ReptilesCelise Taylor
 

Ähnlich wie ETAS_MCQ_14 plants and creatures of dermatologic significance (20)

bio control of cockroach
bio control of cockroachbio control of cockroach
bio control of cockroach
 
My ppt
My pptMy ppt
My ppt
 
Parasitebiology
ParasitebiologyParasitebiology
Parasitebiology
 
Arthropods
ArthropodsArthropods
Arthropods
 
Pathology
PathologyPathology
Pathology
 
Medical entomology lecture ppt
Medical entomology lecture pptMedical entomology lecture ppt
Medical entomology lecture ppt
 
ENTO 301 – MEDICAL & VETERINARY ENTOMOLOGY (Lectures 1-5) ENTO 301 – MEDICA...
ENTO 301 – MEDICAL & VETERINARY ENTOMOLOGY (Lectures 1-5) 	 ENTO 301 – MEDICA...ENTO 301 – MEDICAL & VETERINARY ENTOMOLOGY (Lectures 1-5) 	 ENTO 301 – MEDICA...
ENTO 301 – MEDICAL & VETERINARY ENTOMOLOGY (Lectures 1-5) ENTO 301 – MEDICA...
 
Medical entamology
Medical entamologyMedical entamology
Medical entamology
 
Entomopathogenenic nematodes
Entomopathogenenic nematodesEntomopathogenenic nematodes
Entomopathogenenic nematodes
 
Helminths in adult population in countries.pptx
Helminths in adult population in countries.pptxHelminths in adult population in countries.pptx
Helminths in adult population in countries.pptx
 
2.pptx
2.pptx2.pptx
2.pptx
 
Rickettsiaceae 1
Rickettsiaceae 1Rickettsiaceae 1
Rickettsiaceae 1
 
Human resources section_8-textbook_on_public_health_and_community_medicine
Human resources section_8-textbook_on_public_health_and_community_medicineHuman resources section_8-textbook_on_public_health_and_community_medicine
Human resources section_8-textbook_on_public_health_and_community_medicine
 
Murine Typhus
Murine TyphusMurine Typhus
Murine Typhus
 
Tick Borne Diseases of Public Significance and Integrated Vector Management
Tick Borne Diseases of Public Significance and Integrated Vector ManagementTick Borne Diseases of Public Significance and Integrated Vector Management
Tick Borne Diseases of Public Significance and Integrated Vector Management
 
Flies and mosquito related medical conditions and vector control ppt
Flies and mosquito related medical conditions and vector control pptFlies and mosquito related medical conditions and vector control ppt
Flies and mosquito related medical conditions and vector control ppt
 
Tick borne parasitic infections
Tick borne parasitic infectionsTick borne parasitic infections
Tick borne parasitic infections
 
2010.05.17_FRACP_Insect_Sting_Allergy-2.pdf
2010.05.17_FRACP_Insect_Sting_Allergy-2.pdf2010.05.17_FRACP_Insect_Sting_Allergy-2.pdf
2010.05.17_FRACP_Insect_Sting_Allergy-2.pdf
 
Rickettsia & chlamydia presentation.
Rickettsia & chlamydia presentation.Rickettsia & chlamydia presentation.
Rickettsia & chlamydia presentation.
 
Citrobacter frendii infections in Reptiles
Citrobacter frendii infections in ReptilesCitrobacter frendii infections in Reptiles
Citrobacter frendii infections in Reptiles
 

Mehr von Derma202

Phototherapy treatment protocol
Phototherapy treatment protocolPhototherapy treatment protocol
Phototherapy treatment protocolDerma202
 
Histopathplogical photos
Histopathplogical photosHistopathplogical photos
Histopathplogical photosDerma202
 
Slide study from ETAS
Slide  study from ETASSlide  study from ETAS
Slide study from ETASDerma202
 
Arab board primary exam in dermatology 2012
Arab board primary exam  in dermatology 2012Arab board primary exam  in dermatology 2012
Arab board primary exam in dermatology 2012Derma202
 
Dermatology
DermatologyDermatology
DermatologyDerma202
 
ETAS_MCQ_15 dermatologic and cosmetic surgery
ETAS_MCQ_15 dermatologic and cosmetic surgeryETAS_MCQ_15 dermatologic and cosmetic surgery
ETAS_MCQ_15 dermatologic and cosmetic surgeryDerma202
 
ETAS_MCQ_01 structures of skin
ETAS_MCQ_01 structures of skinETAS_MCQ_01 structures of skin
ETAS_MCQ_01 structures of skinDerma202
 
Derm handbook for medical students and junior doctors 2010
Derm handbook for medical students and junior doctors 2010Derm handbook for medical students and junior doctors 2010
Derm handbook for medical students and junior doctors 2010Derma202
 

Mehr von Derma202 (8)

Phototherapy treatment protocol
Phototherapy treatment protocolPhototherapy treatment protocol
Phototherapy treatment protocol
 
Histopathplogical photos
Histopathplogical photosHistopathplogical photos
Histopathplogical photos
 
Slide study from ETAS
Slide  study from ETASSlide  study from ETAS
Slide study from ETAS
 
Arab board primary exam in dermatology 2012
Arab board primary exam  in dermatology 2012Arab board primary exam  in dermatology 2012
Arab board primary exam in dermatology 2012
 
Dermatology
DermatologyDermatology
Dermatology
 
ETAS_MCQ_15 dermatologic and cosmetic surgery
ETAS_MCQ_15 dermatologic and cosmetic surgeryETAS_MCQ_15 dermatologic and cosmetic surgery
ETAS_MCQ_15 dermatologic and cosmetic surgery
 
ETAS_MCQ_01 structures of skin
ETAS_MCQ_01 structures of skinETAS_MCQ_01 structures of skin
ETAS_MCQ_01 structures of skin
 
Derm handbook for medical students and junior doctors 2010
Derm handbook for medical students and junior doctors 2010Derm handbook for medical students and junior doctors 2010
Derm handbook for medical students and junior doctors 2010
 

Kürzlich hochgeladen

PNEUMOTHORAX AND ITS MANAGEMENTS.pdf
PNEUMOTHORAX   AND  ITS  MANAGEMENTS.pdfPNEUMOTHORAX   AND  ITS  MANAGEMENTS.pdf
PNEUMOTHORAX AND ITS MANAGEMENTS.pdfDolisha Warbi
 
Giftedness: Understanding Everyday Neurobiology for Self-Knowledge
Giftedness: Understanding Everyday Neurobiology for Self-KnowledgeGiftedness: Understanding Everyday Neurobiology for Self-Knowledge
Giftedness: Understanding Everyday Neurobiology for Self-Knowledgeassessoriafabianodea
 
April 2024 ONCOLOGY CARTOON by DR KANHU CHARAN PATRO
April 2024 ONCOLOGY CARTOON by  DR KANHU CHARAN PATROApril 2024 ONCOLOGY CARTOON by  DR KANHU CHARAN PATRO
April 2024 ONCOLOGY CARTOON by DR KANHU CHARAN PATROKanhu Charan
 
Measurement of Radiation and Dosimetric Procedure.pptx
Measurement of Radiation and Dosimetric Procedure.pptxMeasurement of Radiation and Dosimetric Procedure.pptx
Measurement of Radiation and Dosimetric Procedure.pptxDr. Dheeraj Kumar
 
The next social challenge to public health: the information environment.pptx
The next social challenge to public health:  the information environment.pptxThe next social challenge to public health:  the information environment.pptx
The next social challenge to public health: the information environment.pptxTina Purnat
 
ANTI-DIABETICS DRUGS - PTEROCARPUS AND GYMNEMA
ANTI-DIABETICS DRUGS - PTEROCARPUS AND GYMNEMAANTI-DIABETICS DRUGS - PTEROCARPUS AND GYMNEMA
ANTI-DIABETICS DRUGS - PTEROCARPUS AND GYMNEMADivya Kanojiya
 
PERFECT BUT PAINFUL TKR -ROLE OF SYNOVECTOMY.pptx
PERFECT BUT PAINFUL TKR -ROLE OF SYNOVECTOMY.pptxPERFECT BUT PAINFUL TKR -ROLE OF SYNOVECTOMY.pptx
PERFECT BUT PAINFUL TKR -ROLE OF SYNOVECTOMY.pptxdrashraf369
 
Big Data Analysis Suggests COVID Vaccination Increases Excess Mortality Of ...
Big Data Analysis Suggests COVID  Vaccination Increases Excess Mortality Of  ...Big Data Analysis Suggests COVID  Vaccination Increases Excess Mortality Of  ...
Big Data Analysis Suggests COVID Vaccination Increases Excess Mortality Of ...sdateam0
 
Basic principles involved in the traditional systems of medicine PDF.pdf
Basic principles involved in the traditional systems of medicine PDF.pdfBasic principles involved in the traditional systems of medicine PDF.pdf
Basic principles involved in the traditional systems of medicine PDF.pdfDivya Kanojiya
 
Informed Consent Empowering Healthcare Decision-Making.pptx
Informed Consent Empowering Healthcare Decision-Making.pptxInformed Consent Empowering Healthcare Decision-Making.pptx
Informed Consent Empowering Healthcare Decision-Making.pptxSasikiranMarri
 
Presentation on General Anesthetics pdf.
Presentation on General Anesthetics pdf.Presentation on General Anesthetics pdf.
Presentation on General Anesthetics pdf.Prerana Jadhav
 
97111 47426 Call Girls In Delhi MUNIRKAA
97111 47426 Call Girls In Delhi MUNIRKAA97111 47426 Call Girls In Delhi MUNIRKAA
97111 47426 Call Girls In Delhi MUNIRKAAjennyeacort
 
COVID-19 (NOVEL CORONA VIRUS DISEASE PANDEMIC ).pptx
COVID-19  (NOVEL CORONA  VIRUS DISEASE PANDEMIC ).pptxCOVID-19  (NOVEL CORONA  VIRUS DISEASE PANDEMIC ).pptx
COVID-19 (NOVEL CORONA VIRUS DISEASE PANDEMIC ).pptxBibekananda shah
 
SGK HÓA SINH NĂNG LƯỢNG SINH HỌC 2006.pdf
SGK HÓA SINH NĂNG LƯỢNG SINH HỌC 2006.pdfSGK HÓA SINH NĂNG LƯỢNG SINH HỌC 2006.pdf
SGK HÓA SINH NĂNG LƯỢNG SINH HỌC 2006.pdfHongBiThi1
 
Case Report Peripartum Cardiomyopathy.pptx
Case Report Peripartum Cardiomyopathy.pptxCase Report Peripartum Cardiomyopathy.pptx
Case Report Peripartum Cardiomyopathy.pptxNiranjan Chavan
 
Nutrition of OCD for my Nutritional Neuroscience Class
Nutrition of OCD for my Nutritional Neuroscience ClassNutrition of OCD for my Nutritional Neuroscience Class
Nutrition of OCD for my Nutritional Neuroscience Classmanuelazg2001
 
METHODS OF ACQUIRING KNOWLEDGE IN NURSING.pptx by navdeep kaur
METHODS OF ACQUIRING KNOWLEDGE IN NURSING.pptx by navdeep kaurMETHODS OF ACQUIRING KNOWLEDGE IN NURSING.pptx by navdeep kaur
METHODS OF ACQUIRING KNOWLEDGE IN NURSING.pptx by navdeep kaurNavdeep Kaur
 
Wessex Health Partners Wessex Integrated Care, Population Health, Research & ...
Wessex Health Partners Wessex Integrated Care, Population Health, Research & ...Wessex Health Partners Wessex Integrated Care, Population Health, Research & ...
Wessex Health Partners Wessex Integrated Care, Population Health, Research & ...Wessex Health Partners
 
Tans femoral Amputee : Prosthetics Knee Joints.pptx
Tans femoral Amputee : Prosthetics Knee Joints.pptxTans femoral Amputee : Prosthetics Knee Joints.pptx
Tans femoral Amputee : Prosthetics Knee Joints.pptxKezaiah S
 

Kürzlich hochgeladen (20)

PNEUMOTHORAX AND ITS MANAGEMENTS.pdf
PNEUMOTHORAX   AND  ITS  MANAGEMENTS.pdfPNEUMOTHORAX   AND  ITS  MANAGEMENTS.pdf
PNEUMOTHORAX AND ITS MANAGEMENTS.pdf
 
Giftedness: Understanding Everyday Neurobiology for Self-Knowledge
Giftedness: Understanding Everyday Neurobiology for Self-KnowledgeGiftedness: Understanding Everyday Neurobiology for Self-Knowledge
Giftedness: Understanding Everyday Neurobiology for Self-Knowledge
 
April 2024 ONCOLOGY CARTOON by DR KANHU CHARAN PATRO
April 2024 ONCOLOGY CARTOON by  DR KANHU CHARAN PATROApril 2024 ONCOLOGY CARTOON by  DR KANHU CHARAN PATRO
April 2024 ONCOLOGY CARTOON by DR KANHU CHARAN PATRO
 
Epilepsy
EpilepsyEpilepsy
Epilepsy
 
Measurement of Radiation and Dosimetric Procedure.pptx
Measurement of Radiation and Dosimetric Procedure.pptxMeasurement of Radiation and Dosimetric Procedure.pptx
Measurement of Radiation and Dosimetric Procedure.pptx
 
The next social challenge to public health: the information environment.pptx
The next social challenge to public health:  the information environment.pptxThe next social challenge to public health:  the information environment.pptx
The next social challenge to public health: the information environment.pptx
 
ANTI-DIABETICS DRUGS - PTEROCARPUS AND GYMNEMA
ANTI-DIABETICS DRUGS - PTEROCARPUS AND GYMNEMAANTI-DIABETICS DRUGS - PTEROCARPUS AND GYMNEMA
ANTI-DIABETICS DRUGS - PTEROCARPUS AND GYMNEMA
 
PERFECT BUT PAINFUL TKR -ROLE OF SYNOVECTOMY.pptx
PERFECT BUT PAINFUL TKR -ROLE OF SYNOVECTOMY.pptxPERFECT BUT PAINFUL TKR -ROLE OF SYNOVECTOMY.pptx
PERFECT BUT PAINFUL TKR -ROLE OF SYNOVECTOMY.pptx
 
Big Data Analysis Suggests COVID Vaccination Increases Excess Mortality Of ...
Big Data Analysis Suggests COVID  Vaccination Increases Excess Mortality Of  ...Big Data Analysis Suggests COVID  Vaccination Increases Excess Mortality Of  ...
Big Data Analysis Suggests COVID Vaccination Increases Excess Mortality Of ...
 
Basic principles involved in the traditional systems of medicine PDF.pdf
Basic principles involved in the traditional systems of medicine PDF.pdfBasic principles involved in the traditional systems of medicine PDF.pdf
Basic principles involved in the traditional systems of medicine PDF.pdf
 
Informed Consent Empowering Healthcare Decision-Making.pptx
Informed Consent Empowering Healthcare Decision-Making.pptxInformed Consent Empowering Healthcare Decision-Making.pptx
Informed Consent Empowering Healthcare Decision-Making.pptx
 
Presentation on General Anesthetics pdf.
Presentation on General Anesthetics pdf.Presentation on General Anesthetics pdf.
Presentation on General Anesthetics pdf.
 
97111 47426 Call Girls In Delhi MUNIRKAA
97111 47426 Call Girls In Delhi MUNIRKAA97111 47426 Call Girls In Delhi MUNIRKAA
97111 47426 Call Girls In Delhi MUNIRKAA
 
COVID-19 (NOVEL CORONA VIRUS DISEASE PANDEMIC ).pptx
COVID-19  (NOVEL CORONA  VIRUS DISEASE PANDEMIC ).pptxCOVID-19  (NOVEL CORONA  VIRUS DISEASE PANDEMIC ).pptx
COVID-19 (NOVEL CORONA VIRUS DISEASE PANDEMIC ).pptx
 
SGK HÓA SINH NĂNG LƯỢNG SINH HỌC 2006.pdf
SGK HÓA SINH NĂNG LƯỢNG SINH HỌC 2006.pdfSGK HÓA SINH NĂNG LƯỢNG SINH HỌC 2006.pdf
SGK HÓA SINH NĂNG LƯỢNG SINH HỌC 2006.pdf
 
Case Report Peripartum Cardiomyopathy.pptx
Case Report Peripartum Cardiomyopathy.pptxCase Report Peripartum Cardiomyopathy.pptx
Case Report Peripartum Cardiomyopathy.pptx
 
Nutrition of OCD for my Nutritional Neuroscience Class
Nutrition of OCD for my Nutritional Neuroscience ClassNutrition of OCD for my Nutritional Neuroscience Class
Nutrition of OCD for my Nutritional Neuroscience Class
 
METHODS OF ACQUIRING KNOWLEDGE IN NURSING.pptx by navdeep kaur
METHODS OF ACQUIRING KNOWLEDGE IN NURSING.pptx by navdeep kaurMETHODS OF ACQUIRING KNOWLEDGE IN NURSING.pptx by navdeep kaur
METHODS OF ACQUIRING KNOWLEDGE IN NURSING.pptx by navdeep kaur
 
Wessex Health Partners Wessex Integrated Care, Population Health, Research & ...
Wessex Health Partners Wessex Integrated Care, Population Health, Research & ...Wessex Health Partners Wessex Integrated Care, Population Health, Research & ...
Wessex Health Partners Wessex Integrated Care, Population Health, Research & ...
 
Tans femoral Amputee : Prosthetics Knee Joints.pptx
Tans femoral Amputee : Prosthetics Knee Joints.pptxTans femoral Amputee : Prosthetics Knee Joints.pptx
Tans femoral Amputee : Prosthetics Knee Joints.pptx
 

ETAS_MCQ_14 plants and creatures of dermatologic significance

  • 1. Plants and Creatures of Dermatologic Significance 1) Name this creature: A. Demodex folliculorum B. Ornithonyssus sylviarum C. Pediculus capitis D. Pediculus humanusCorrect Choice E. Pthirus pubis This is the body louse, or Pediculus humanus var. corporis. The pubic louse (pthirus pubis) is shorter, and squatter. The head louse (pediculus capitis) has a narrow body 2) When doing patch testing for compositae sensitivity, which of the following substances is used? A. Tuliposide A B. Rhus C. Sesquiterpene lactone mix Correct Choice D. Colophony E. Pentadecacatechols Colophony cross reacts with turpentine resin, hairpiece adhesive, mascara, dental cement and others. Pentadecacatechols are in the rhus family and are found in poision ivy, oak and sumac in addition to cross reacting with mango and cashew. Tuliposide A is found in Peruvian lily (alstomeria), and tulip 3) A previously healthy 25-year-old man presents to the ER 8 hours after experiencing an extremely painful spider bite. The bite site is erythematous and indurated, without necrosis or eschar. The patient is vomitting and experiencing chills, violent cramps, and abdominal pain and rigidity. The surgical consultant suspects an acute abdomen. You correctly propose that: A. The patient has a likely Lycosidae bite and needs supportive care B. The patient has a likely Loxosceles bite and should go to the OR for excision and debridement C. The patient needs anti-venom after a likely Phidippus formosus bite D. The patient has likely experienced a hobo spider bite E. The patient needs anti-venom after a likely Lactrodectus biteCorrect Choice The patient has signs and symptoms a black widow spider bite, caused by Lactrodectus. Anti-venom may be helpful up to 90 hours after the bite 4) Match the photo to the plant identified: A. Poison oak B. Poison sumac Correct Choice C. Mango leaves D. Gingko E. Poison ivy Poison sumac leaves grow in groupings of 7-13 with a single leaf at the tip. 1
  • 2. 5) All of the following are food mites except: A. DermatophagoidesCorrect Choice B. Acarus C. Glyciphagus D. All of the listed answers are food mites E. Tyrophagus Acarus (grain mite), Tyrophagus (grocery mite), and Glyciphagus (cheese mite) are all food mites that ingest foodstuffs. They can produce papular urticaria or vesicopapular eruptions. Dermatophagoides is the dust mite, and causes environmental allergic reactions. 6) Which of the following statements regarding Megalopyge opercularis is correct? A. Are also called "bed bugs" by lay people B. These creatures can be vectors of many diseases from typhus to plague C. These creatures, found in Hawaii, are nocturnal carnivores that produce painful wounds by discharging venom D. Contact with this creature often causes immediate pain, urticaria, and erythematous papules or purpura in a tram-track patternCorrect Choice E. Bites are usually asymptomatic, initially, but then form linear purpuric macules Megalopyge opercularis (puss caterpillar) is one of the most widespread causes of caterpillar dermatitis. Contact with this creature often causes immediate pain, urticaria, and erythematous papules or purpura in a tram-track pattern. Bites of Cimicidae (bedbugs) are usually asymptomatic, initially, but then form linear purpuric macules. Fleas can be vectors of many diseases from typhus to plague. Chilopoda (centipedes) are nocturnal carnivores that produce painful wounds by discharging venom. Scolopendra is a species of centipede found in Hawaii 7) The venom from which of the following creatures contains phospholipase A? A. Cimicidae B. Ctenocephalides canis C. Theraphosidae D. HoneybeeCorrect Choice E. Solenopsis invicta Honeybees (Hymenoptera) leave a barbed ovipositor and paired venom sacs impaled into their victim. The honeybee dies after stinging, but other hymenoptera do not. The venom from the honeybee contains phospholipase A. Theraphosidae (tarantulas) are of dermatologic importance due to the presence of urticating hairs. Cimicidae (bedbugs) cause initially asymptomatic bites that then form linear, purpuric, pruritic macules and erythematous papules. Ctenocephalides canis is a flea that causes frequent infestations on dogs. Solenopsis invicta is the imported fire ant, and attacks in groups. Solenopsis inject a venom that contains a hemolytic factor, solenopsin D 8) Unilateral swelling of the eyelid at the site of the bite of the assassin bug is called what? A. Rothschild's sign 2
  • 3. B. Romana's signCorrect Choice C. Russell's sign D. Raynaud's sign E. Unilateral Chagas' disease Rothschild's sign is alopecia of the lateral 1/3 of the eyebrows secondary to hypothyroidism. Russell's sign is the development of callouses on the dorsal dominant hand. Raynaud's sign is the development of acrocyanosis 9) A butcher’s nodule is caused by which of the following? A. HPV 2, 7 Correct Choice B. HPV 16, 18 C. HPV 1 D. HPV 24 E. HPV 13, 32 HPV 24 causes common warts. HPV 16 and 18 are found in genital warts; 1 is for plantar warts and types 13 and 32 are seen in Heck’s Disease (focal epithelial hyperplasia 10) Immunologic contact urticaria is frequently caused by: A. None of these answers are correct B. Plants of the Urticaceae family C. Hevea brasiliensisCorrect Choice D. All of these answers are correct E. Plants of the Euphorbiacea family Contact urticaria occurs after direct contact, and can be immunologic (IgE) or non-immunologic (toxin-mediated). Type I hypersensitivity, or IgE mediated reactions, require previous sensitization. Frequent causes of immunologic eruptions include latex proteins (Hevea brasiliensis). Plants such as stinging nettles (Urticaceae family) and spurge nettle (Euphorbiaceas) cause nonimmunologic urticaria, which occurs without previous sensitization in any host. 11) Phytophotodermatitis is caused by which of the following wavelengths of light? A. 290-320 nm B. 320-400 nm Correct Choice C. 100-200 nm D. 200-290 nm E. 311 nm UVA rays are responsible for the phototoxic reaction which occurs with phytophotodermatitis 12) Identify this caterpillar: A. Greenback caterpillar B. Hag moth caterpillar 3
  • 4. C. Io caterpillar D. Puss caterpillar E. Saddleback caterpillarCorrect Choice This is the saddleback caterpillar, as it appears that the caterpillar is wearing a saddle 13) Which of the following is false regarding this image? A. This is a glossina fly B. This transmits trypanosomes C. A large abscess may occur following a biteCorrect Choice D. This transmits sleeping sickness E. This is the tsetse fly Cutaneous reactions tend to be minimal from the bite of the tsetse, or glossina, fly which transmits trypanosomes and sleeping sickness 14) Flying squirrels can carry many infections, including which of the following? A. all of these infections are correctCorrect Choice B. Toxoplasma gondii C. Staphlyococcus sp D. none of these infections are correct E. Epidemic typhus Flying squirrels can carry many infections, including Toxoplasma gondii, Staphylococcus sp., and Rickettsia prowazekii (via the body lous, causing epidemic typhus 15) A patient who experiences an allergic contact dermatitis in reaction to an extract from the Pinus palustris tree may also be sensitized to: A. None of these answers are correct B. Varnish C. Mascara D. Rosin E. All of these answers are correctCorrect Choice Colophony, derived from the Pinus palustris tree, may cross react with rosin, medications, mascara, adhesives, bandages, varnish, wax, paper products, and dental cement 16) A butcher presents with numerous verrucous papules of the hands and fingers. Based on her occupation, you consider a diagnosis of infection with which HPV types? A. HPV 2 and 7Correct Choice B. HPV 1 and 2 C. HPV 6 and 8 D. HPV 16 and 18 4
  • 5. E. HPV is not associated with this condition Meat, fish, and poultry handlers often suffer from hand and finger verrucae, usually caused by human papilloma virus types 2 and 7. 17) This snake, the coral snake, is a member of what family? A. ElapidaeCorrect Choice B. Lepidoptera C. Theraphosidae D. Crotalidae E. Apiaceae The Crotalidae family includes rattlesnakes and copperheads. Lepidoptera are caterpillars. Theraphosidae is the scientific name for tarantulas. Apiaceae (Umbelliferae) is a plant family which includes parsley, celery and fennel. Elapidae is the correct family of snakes, including coral snakes 18) Which of the following plants contains thiocyanates that can cause an irritant dermatitis? A. Radish B. All of these answers are correctCorrect Choice C. Plants of the Brassicaceae family D. Mustard E. Garlic Thiocyanates are found in garlic (Alliaceae family), mustard, and radish (Brassicaceae family 19) This tick transmits: A. Babesiosis B. Erlichiosis C. Rocky mountain spotted feverCorrect Choice D. Relapsing fever E. Lyme This is Dermacentor variabilis which transmits Rocky Mountain Spotted Fever as well as Tularemia 20) Which family of plants is most commonly implicated in phytophotodermatitis caused by furocoumarins? A. Moraceae B. Papilonaceae C. ApiaceaeCorrect Choice D. Compositae E. Rutaceae The Apiaceae (formerly umbelliferae) family is the most common cause of phytophotodermatitis. The members of this family include cow parsley, celery, wild parsnip, false bishop's weed, giant hogweed, angelica, meadow grass, fennel, wild carrot, caraway, and coriander 5
  • 6. 21) The reaction caused by this plant is mediated by: A. IgE B. Eosinophils C. Langerhans cellsCorrect Choice D. Mast cells E. Histamine This is ragweed, which causes allergic contact dermatitis. Mast cells, IgE and histamine are characteristic of contact urticaria. Allergic contact dermatitis is caused by langerhans cells presenting antigen to CD4+ lymphocytes leading to a cell-mediated delayed hypersentitivity reaction 22) Which of the following mites is a cause of "walking dandruff" in dogs and cats? A. All of these answers are correct B. None of these answers are correct C. Allodermanyssus sanguineus D. Acarus E. CheyletiellaCorrect Choice Cheyletiella mites are harbored by dogs and cats and cause walking dandruff. The pet is asymptomatic, but people handling the pet may experience pruritus when mites feed on skin. Diagnosis is by microscopic examination of cellophane tape applied to the pet's skin. Acarus is the grain mite, and causes baker's itch. Allodermanyssus sanguineus is the house mouse mite, and the vector of rickettsial pox 23) Which of the following statements regarding Theraphosidae is correct? A. All of these statements are correctCorrect Choice B. These creatures possess urticating hairs that are of dermatologic significance C. These creatures can cause visual loss by causing a chronic granulomatous reaction (ophthalmia nodosa) D. None of these statements are correct E. Bites do not generally produce any systemic toxicity Theraphosidae (tarantulas) are large, brown to black, hairy spiders frequently found in the southwest. They are of dermatologic importance due to urticating hairs. Tarantula bites generally do not produce any systemic toxicity. The spider releases hairs in the direction of the perceived attacker; hairs can penetrate the skin as deeply as the reticular dermis. If they injure the cornea, they can cause a chronic granulomatous reaction (ophthalmia nodosa) and loss of vision 24) A veterinarian presents with an edematous pustule on the right arm, with nodules along the lymphatic drainage route. You consider a diagnosis of glanders. What treatment do you recommend? A. Surgical excision followed by antibiotic treatment with streptomycin combined with tetracyclineCorrect Choice 6
  • 7. B. Reassurance. No treatment is necessary, as infection is self-limited and should resolve within 6 weeks C. Surgical excision followed by antibiotic treatment with rifampin and ethambutol D. Surgical excision E. Antibiotic treatment with streptomycin combined with tetracycline Farcy, or glanders, is caused by infection with the gram-negative rod Pseudomonas mallei. It most frequently infects horses, donkeys, and mules; humans can occasionally also be infected. An edematous nodule, pustule, or vesicle develops at the inoculation site. Nodules are often present along the lymphatic drainage route and are called "farcy buds." Importantly, nasal ulceration and septum perforation can be caused by glanders. A chronic form leads to deep, painful abscesses. Treatment is with surgical excision of the lesion followed by streptomycin combined with tetracycline. 25) Ananas comosus causes an irritant dermatitis attributed to which irritant? A. Phorbol esters B. Thiocyanates C. Capsaicin D. Ranunculin E. BromelinCorrect Choice Ananas comosus is the pineapple. Pineapples contain the irritants bromelin (a proteolytic enzyme) and calcium oxalate (which causes fissures that grant bromelin access to dermal vessels). Capsaicin is found in chili peppers. Phorbol esters are found in spurges, crotons, poinsettas, and machineel trees. Thiocyanates are found in garlic, mustard, and radishes. Ranunculin is found in buttercups 26) This is a: A. Moth caterpillar B. Woolybear caterpillarCorrect Choice C. Brownbear caterpillar D. Puss caterpillar E. Furrybear caterpillar This is a woolybear caterpillar 27) Onions and chives: A. Are members of the family Anacardiacea B. Contain diallyl disulfideCorrect Choice C. Cross-react with artichoke in sensitized patients D. Contain pentadecylcatechol E. Are members of the same family as Gingko Onions, garlic, and chive are members of the family Alliaceae and contain diallyl disulfide (also allylpropyl disulfide, allicin 28) Poison sumac is: 7
  • 8. A. A source of colophony B. The common name for Myroxylon balsamum C. A source of tuliposide A D. The common name for Pinus palustris E. A member of the Toxicodendron genusCorrect Choice Poison ivy, oak, and sumac are members of the Anacardiacea family, Toxicodendron genus. Alstromeria (Peruvian lily) is the source of tuliposide A. Myroxylon balsamum is the source of Balsam of Peru. The Pinus palustris tree is the source of colophony 29) Cantharadin is derived from: A. Ranunculaceae B. Compositae C. Lytta vesicatoriaCorrect Choice D. Pentadecacatechol E. Solanaceae Ranunculaceae (buttercup) causes irritant dermatitis. Pentadecacatechol is the rhus antigen found in poison ivy, oak and sumac. Solanaceae (chili pepper) contains capsaicin. Pyrethrin is derived from compositae (chrysanthemum flower). Lytta vesicatoria (spanish fly) is a blister beetle from which cantharadin is made 30) Which of these mites is the most likely cause of a pruritic papulovesicular eruption in a baker? A. Dermanyssus B. Dermatophagoides C. none of the above D. Cheyletiella E. AcarusCorrect Choice Acarus (grain mite) causes so-called "baker's itch." Dermatophagoides (dust mite) causes allergic reactions, Dermanyssus (fowl mite) causes equine encephalitis, and Cheyletiella causes walking dandruff in dogs and cats 31) Lipase is a notable active component of the venom of which of the following spiders? A. Loxosceles B. Hidippus C. ChiracanthiumCorrect Choice D. Lactrodectus E. Lycosidae The venom of Chiracanthium (sac spiders) contains lipase. The venom of Lycosidae (wolf spiders) is notable for histamine; Lactrodectus (black widow spiders) for alpha lactrotoxin; Hidippus (jumping spiders) for hyaluronidase; and Loxosceles (brown recluse spiders) for sphingomyelinase-D. 32) The reaction in this picture is most likely a result of: 8
  • 9. A. UV exposure Correct Choice B. Child abuse C. Irritant response from garlic D. Toxicodendron bullous reaction E. Nickel This is an example of phytophotodermatitis. This is a reaction between furocoumarins followed by exposure to UVA 33) Furocoumarins, such as 5-methoxypsoralen or 8-methoxypsoralen: A. Cause a type IV delayed hypersensitivity reaction after exposure to all wavelengths of UV light B. Cause a phototoxic reaction after contact with the skin, followed by exposure to ultraviolet light in the 320-400nm rangeCorrect Choice C. Cause a phototoxic reaction after contact with the skin, followed by exposure to ultraviolet light in the 280-320nm range D. Cause a photoallergic reaction after contact with the skin, followed by exposure to ultraviolet light in the 320-400nm range E. Cause a photoallergic reaction after contact with the skin, followed by exposure to ultraviolet light in the 280-320nm range Unlike photoallergic reactions, which involve the immune system (type IV or delayed hypersensitivity), phytophotodermatitis is non-immunologic. Photoallergic reactions are more frequently caused by other, non-plant sources. Furocoumarins, such as 5-methoxypsoralen or 8- methoxypsoralen, cause a phototoxic reaction after contact with the skin, followed by exposure to ultraviolet light in the UVA range (320 to 400nm). 34) A 47 year old park ranger developed a diffuse, erythematous, eczematous eruption after fighting a forest fire. What is the most likely allergen causing this airborn contact dermatitis? A. Alphamethylene B. Pyrethrins C. D-usnic acidCorrect Choice D. Psoralen E. Diallyl disulfide Lichen causes airborne allergic contact dermatitis in lumberjacks, forest workers, and people exposed to firewood, funeral wreaths, and masculine fragrances. The primary allergen is d-usnic acid 35) Which disease is transmitted by this insect? A. Chagas diseaseCorrect Choice B. Onchocerciasis C. African sleeping sickness D. Visceral leishmaniasis E. Dracunculiasis 9
  • 10. This a picture of the reduviid bug, also called the kissing bug or assassin bug. It is the vector for American Trypanosomiasis (Chagas disease), which is caused by Trypanosoma cruzii. The reduviid bug prefers to bite at mucocutaneous junctions 36) Which of the following plants are members of the Solanaceae family? A. Prickly pear B. Radish C. Garlic D. Crotons E. Chili pepperCorrect Choice Chili peppers belong to the Solanaceae family. Prickly pears (Opuntia vulgaris), crotons (Euphorbiaceae family), garlic (Alliaceae family), and radishes (Brassicaceae family) are not members of this family 37) Leishmania donovani is transmitted by: A. Glossina B. Callitroga americana C. Attagenus megatoma D. PhlebotomusCorrect Choice E. Anopheles Leishmania donovani is transmitted by Phlebotomid sandflies. Callitroga americana is the most important cause of cutaneous myiasis in the United States. Glossina, the tsetse fly, transmits sleeping sickness. Attagenus megatoma is a carpet beetle, the larvae of which can cause an allergic contact dermatitis. Anopheles mosquitoes transmit malaria. 38) Sharp hairs on plants of the Urticaceae family contain which of the following toxins that are released into the skin causing rapid edema, pruritus, and burning? A. Serotonin B. Histamine and acetylcholine C. Acetylcholine D. All of these answers are correctCorrect Choice E. Histamine Sharp hairs on plants such as stinging nettles (Urticaceae family) contain toxins (histamine, serotonin, and acetylcholine) which are released into the skin causing rapid edema, pruritus, and burning 39) Which of the following may NOT occur after contact with the organism in the photo? A. Loss of vision B. Purpura in a tram-track pattern C. Urticaria D. Pruritus 10
  • 11. E. Tram-track calcification on skull x-ray Correct Choice The most common response is erythema, pruritus and urticaria. Purpura and hemorrhage in a tram- track or ladder pattern may be seen as well. Ophthalmia nodosa may occur if hairs injure the cornea. Tram-track calcification on skull films is seen in Sturge-Weber syndrome 40) A patient comes to you with complaints of numerous erythematous pruritic papules under her bathing suit after swimming in the ocean. What is the most likely diagnosis? A. Seabather’s eruption Correct Choice B. Swimmer’s itch C. Nematocyst stings D. Hydrozoa contact E. Hot tub folliculitis Sea bather’s eruption is seen after ocean exposure and can be intensely pruritic. It is due to jellyfish larvae (Linuche unguiculata) and can be prevented by rinsing off soon after coming out of the water. Swimmer’s itch is a cercarial dermatitis caused by avian schistosomes which occurs after freshwater exposure. It is usually seen on exposed areas. Portuguese man of war (hydrozoa) stings can lead to hemorrhagic lesions with vesiculations 41) Which of the following plant families commonly causes phytophotodermatitis? A. Rubiaceae B. Ranunculaceae C. Apiaceae Correct Choice D. Compositae E. Urticaceae The Apiaceae family (formerly Umbelliferae) includes parsley, celery, parsnip, hogweed, and fennel, all potential causes of phytophotodermatitis 42) When bitten with the spider in the photograph, the active agent in the venom is: A. Sphingomyelinase-D B. Alpha lactrotoxin Correct Choice C. Lipase D. Hyaluronidase E. Histamine This is a black widow spider which can be identifiec by its shiny black body with characteristic white hourglass marking on the abdomen 43) The vector for Babesia microti is also a vector for: A. Rickettsia rickettsii B. None of these answers are correct C. Francisella tularensis 11
  • 12. D. Borrelia burgdorferiCorrect Choice E. All of these answers are correct The vector for Babesia microti, the pathogenic organism of babesiosis, is Ixodes dammini, which is also a vector for Borrelia burgdorferi, the pathogenic organism of Lyme borreliosis. Francisella tularensis (the pathogenic organism of tularemia) is transmitted by Dermacentor andersoni and Dermacentor variabilis. Rickettsia rickettsii (the pathogenic organism of Rocky Mountain Spotted Fever) is transmitted by Dermacentor andersoni, Dermacentor variabilis, and Amblyomma americanum 44) Allergic contact dermatitis is a: A. Type III hypersensitivity B. Type II hypersensitivity C. Type I hypersensitivity D. Undetermined hypersensitivity E. Type IV hypersensitivity Correct Choice NEEDS EXPLANATIONS 45) Match the photo to the plant identified: A. Mango leaves B. Poison ivy C. Gingko D. Poison oak Correct Choice E. Poison sumac Poison oak is identified by leaf shape - irregular lobing and varied leaf size 46) The insect pictured is: A. A vector of relapsing fever B. The cause of maculae cerulea C. A vector of Lyme borreliosisCorrect Choice D. Ornithonyssus E. A vector of babesiosis The insect pictured is a female Amblyomma americanum tick. This tick can serve as a vector for Lyme borreliosis and Rocky Mountain Spotted Fever. Relapsing fever is transmitted by Ornithodoros moubata. Babesiosis is trasnmitted by Ixodes dammini. Maculae cerulea are seen in severe cases of crab lice. Ornithonyssus is the fowl mite 47) Which of the following is the most common cause of dermatitis in florists? A. Sesquiterpene lactones B. Calcium oxalate Correct Choice C. Primin 12
  • 13. D. Tuliposide A E. Mechanical dermatitis Calcium oxalate is present in the bulbs and stems of Narcissus (daffodil 48) Snake bite antivenin is derived from: A. Pooled IVIg B. Horse antibodyCorrect Choice C. Human antibody D. Mouse antibody E. Antigen extracted directly from the attacking snake Following envenomation, antivenins are derived from horses 49) All of the following plants commonly cause a phytophotodermatitis except: A. Celery B. Ficus carica C. Hawaiian lei flowers D. Myroxylon balsamumCorrect Choice E. All of these plants commonly cause a phytophotodermatitis Myroxylon balsamum is the source of Balsam of Peru, a common sensitizer of allergic contact dermatitis. Hawaiian lei flowers, Ficus carica (fig tree), and celery all are known to commonly cause phytophotodermatitis 50) On slide preparation, many organisms were noted on this patient, as well as occasional blue macules on the buttocks. He most likely has: A. Chigoe fleas B. Pediculus capitis infestation C. Pthirus pubis infestationCorrect Choice D. Sarcoptes scabiei infestation E. Pediculus humanus infestation It is often very easy to find crab lice on physical exam in contrast with scabies. Head lice infestations are usually limited to the scalp. Body lice are not often found on the patient, more commonly in clothing seams. Maculae caerulea can be seen in widespread involvement as in this case 51) Which of the following plants are members of the family Rutaceae? A. Burning bush B. Rue C. All of these answers are correctCorrect Choice D. None of these answers are correct 13
  • 14. E. Bergamot orange The family Rutaceae includes lime, rue, burning bush, bergamot orange, and Hawaiian lei flowers. Plants from this family commonly cause phytophotodermatitis 52) The toxin holothurin, released by which of the following marine creatures, can cause conjunctivitis in exposed swimmers? A. Sea cucumberCorrect Choice B. None of these answers are correct C. Chironex fleckeri D. Edwardsiella lineate E. Linuche unguiculata Sea cucumbers are cucumber-shaped bottom-dwellers that can cause conjunctivitis due to release of a toxin called holothurin to which nearby swimmers can be exposed. The larva of Linuche unguiculata (thimble jelly fish) and Edwardsiella lineate (sea anemone) are causes of seabather's eruption (or sea lice). Chironex fleckeri is the box jellyfish; its stings are extremely toxic and may lead to shock and death, especially in small adults and children 53) Cantharidin creates a vesicle by which of the following mechanisms? A. Subcorneal layer separation B. Instability of the hemidesmosome C. Antibodies to desmosomal proteins D. Local recruitment of neutrophils E. Disruption of keratinocyte-keratinocyte adhesion Correct Choice Desmosomal adhesions are interrupted by proteases with eventual tonofilament detachment 54) Patients with an allergy to the compositae family may have an allergic reaction to which of the following drugs? A. Pyruvic acid B. Pyrethrins Correct Choice C. Psoralens D. Propylene glycol E. Pseudoephedrine Pyrethrins are derived from chrysanthemum flowers. Patients with allergy to these flowers or ragweed may experience allergic symptoms after using pyrethrins 55) This patient complains of intense itching at this site. KOH examination is negative. She denies any recent travel, however does take her 2 year old to the local playground where there is a sandbox. Which of the following is appropriate? A. Give her clobetasol ointment B. Slowly extract the organism C. Inform the patient she is contagious and should isolate herself from her children 14
  • 15. D. Treat with po ivermectin Correct Choice E. Obviously this is dermatophyte despite a negative KOH. Treat with topical antifungals This is hookworm, or cutaneous larva migrans; systemic ivermectin or topical thiobendazole would be appropriate treatments. The causative organism is the larvae of Ancylostoma braziliensis or A. canium 56) This creature can do the following EXCEPT: A. Cause a painful sting B. Transmit Yersinia pestisCorrect Choice C. Cause anaphylaxis D. Be used as a natural control for the tick population E. Build large characteristic mounds This is a fire ant. In some parts of the world, they are used as a natural tick control as they are scavengers and consumers of tick eggs. They can cause anaphylaxis in an allergic person. They do not transmit disease. The mounds are characteristic in appearance 57) Babesia microti: A. Is transmitted by Ixodes dammini B. All of these answers are correctCorrect Choice C. None of these answers are correct D. Is an intracellular parasite E. Is pathogenic in an infection that is endemic on Martha's Vineyard Babesiosis is caused by an intracellular RBC parasite, Babesia microti. It is transmitted by the larvae of Ixodes dammini. This disease is endemic in eastern Long Island, Martha's Vineyard, and Nantucket, and carries an increased risk in those with T-cell depression or after splenectomy. The condition is associated with fever, drenching sweats, myalgias, and hemolytic anemia 58) Which chemical is present in large quantities of this insect's venom? A. Lipase B. Hyaluronidase C. Sphingomyelinase D D. Alpha lactrotoxin E. HistamineCorrect Choice The spider shown is the Wolf spider (Lycosidae). This Australian spider's bites contains large amounts of histamine. Bites are painful with edema, erythema and subsequent lymphangitis 59) Which of the following is false? A. Divers are at risk of contacting these creatures B. Only pigmented varieties pose any riskCorrect Choice C. These spines may contain toxins 15
  • 16. D. This is a sea urchin E. Granulomas can form around the spines if not removed This is a sea cucumber. A small fraction of spines contain toxins which affect humans. Divers may step on these creatures which is quite painful. Spines should be removed to avoid foreign body reactions 60) Which of the following mites is also known as the chigger and often causes intense pruritus on the ankles, legs, or belt line? A. Glyciphagus B. Allodermanyssus sanguineus C. Trombicula alfreddugesi Correct Choice D. Demodex E. Ornithonyssus Trombicula alfreddugesi (chigger mite, harvest mites) frequently causes intense pruritus on the ankles, legs, or belt line. Contact occurs during summer and fall. In sensitized individuals, papular urticaria, vesiculation, or a granulomatous reaction with fever and lymphadenopathy may occur. Allodermanyssus sanguineus (house mouse mite) is the vector of rickettsial pox. Demodex folliculorum is a mite that can live within the sebaceous glands of hair follicles and may be important in some subsets of patients with acne rosacea, as well as patients with HIV infection and folliculitis. Glyciphagus (cheese mite) is the cause of grocer's itch. Ornithonyssus (fowl mite) can carry Western equine encephalitis 61) Contact urticaria is a: A. Type II hypersensitivity B. Type I hypersensitivity Correct Choice C. Type III hypersensitivity D. Undetermined hypersensitivity E. Type IV hypersensitivity NEEDS EXPLANATIONS 62) Which of these spiders is bright green with red spots, with black spines on its legs? A. Phidippus formosus B. Loxosceles reclusa C. Latrodectus mactans D. Peucetia viridansCorrect Choice E. Centruroides sculpturatus Peucetia viridans (green lynx spider) is common in the southern United States. It is bright green with red spots; the legs have black spines. The bite is painful, with tenderness and pruritus. Phidippus formosus (jumping spider) is an aggressive spider with dark body hairs and various white patterns. They are the most common biting spiders in the United States. Loxosceles reclusa (brown recluse) is a tan spider with a violin-shaped marking on the abdomen. The bite may be painless, however the patient may develop erythema, a vesicle, and eventual necrosis. Latrodectus mactans (black widow) is a large, black, shiny spider with an hourglass shaped white marking on the 16
  • 17. abdomen. Bites are acutely painful, and may swell but do not cause necrosis; the venom contains neurotoxins. Centruroides sculpturatus is a primary scorpion species in the U.S 63) Mechanical irritant dermatitis can be caused by: A. GlochidsCorrect Choice B. Bromelin C. Stinging nettles D. Capsacin E. All of these answers are correct Mechanical irritant dermatitis can be caused by the direct effects of thorns and barbs. Small glochids (barbed hairs or brisles) or large thorns (on cacti for example) can become imbedded into the skin causing injury and resultant dermatitis. Secondary infection can be caused by microorganism inoculation 64) Which of the statements regarding Tegenaria agrestis is correct? A. This creature is the leading cause of necrotic arachnidism in several states of the Pacific NorthwestCorrect Choice B. None of these answers are correct C. Contact with this creature results in erythematous papules, hemorrhage, or purpura in a classic "tram-track" pattern D. Stings from this scorpion can produce cardiovascular complications E. This ant injects a venom that contains a hemolytic factor, solenopsin D Tegenaria agrestis (the Hobo spider) is the leading cause of necrotic arachnidism in several states of the Pacific Northwest. These spiders are large, with a herringbone-striped pattern on the abdomen. Contact with caterpillars (Lepidoptera) may produce lesions in a classic "tram-track" pattern. Solenopsis (fire ants) ants inject a venom that contains a hemolytic factor, solenopsin D, that causes a release of histamine and other vasoactive amines from mast cells 65) Insects from the species pictured: A. Trasmit yellow fever B. Transmit malaria and yellow fever C. Transmit dengue fever D. Trasmit yellow fever and dengue feverCorrect Choice E. Transmit malaria The insect pictured is an Aedes mosquito. Aedes transmits yellow fever and dengue fever. Anopheles mosquitoes transmit malaria 66) A 52 year-old gardener presents with innumerable linear hyperpigmented streaks across the chest and arms. He states that several weeks previously he had a painful, erythematous, blistering eruption at the same site. The most likely botanical cause for this eruption is: A. Plants of the Alliaceae family B. Lime 17
  • 18. C. HogweedCorrect Choice D. Peruvian lily E. Ragweed The gardener has so-called "strimmer dermatitis." The use of a weed-whacker may result in a spray of weeds (such as cow parsley, wild chervil, and hogweed) leading to this phytophotodermatitis on exposed areas such as the chest and arms 67) Identify this plant: A. Alstromeria B. Toxicodentron C. Myroxylon balsamum D. PrimroseCorrect Choice E. Ragweed This is primrose (primulaceae) a cause of allergic contact dermatitis 68) Necrosis may be seen in which of the following: A. Allergic contact dermatitis B. Irritant contact dermatitis Correct Choice C. Dermatographism D. Contact urticaria E. Acute urticaria Necrosis may be seen in severe irritant reactions. The other reactions do not involve necrosis 69) Which creature is a human flea that can also be seen on dogs? A. Liponyssoides sanuineus B. Pulex irritansCorrect Choice C. Lytta vesicatoria D. Ctenocephalides canis E. Pediculus humanus Pulex irritans is the human flea; it can also be seen on dogs. More frequent infestations on domestic animals are by Ctenocephalides felis and canis (on cats and dogs respectively). Lytta vesicatoria is a blister beetle from which cantharadin is derived. Pediculus humanus var. corporis is the human body louse. 70) Which of the following is a complication of Thermage treatment? A. Hyperpigmentation B. LipoatrophyCorrect Choice C. Keloid formation D. Granuloma formation 18
  • 19. E. Herpes simplex infection Thermage (ThermaCool) is a nonablative radiofrequency-based system which uses volumetric heating to induce tightening of the skin and dermal remodeling. Lipoatrophy is a late-occurring potential complication of this technology 71) Which of the following is not a hallmark of allergic contact dermatitis? A. Eosinophils B. Neutrophils Correct Choice C. Langerhans cells D. CD4+ Lymphocytes E. Vesicles Langerhans cells present the antigens to CD4+ lymphocytes in the dermis and after traveling through lymphatics to regional lymph nodes. Eosinophils may also be present. Neutrophils are not characteristic of this reaction 72) The female Sarcoptes scabiei var. hominis mite can lay up to 100 eggs. It has an ovoid, flat body and how many short legs? A. 10 B. 6 C. None of these answers are correct D. 12 E. 8Correct Choice The female Sarcoptes scabiei var. hominis mite has an ovoid, flat body and 8 short legs 73) Which snake is part of the Crotalidae family? A. Garden snake B. Coral snake C. RattlesnakeCorrect Choice D. Cobra E. Water snake The Crotalidae family includes rattlesnakes and copperheads. The coral snake is part of the elapidae family 74) A 22 year-old man presents with an irregular laceration on the dorsal metacarpals following a fist fight in which he struck someone in the mouth. The tissue surrounding the wound is erythematous, edematous, indurated, warm, and tender. There is some purulence of the wound. You consider infection with: A. Erysipelothrix rhusiopathiae B. Eikenella corrodensCorrect Choice C. Pasteurella multocida 19
  • 20. D. Pasteurella canis E. Capnocytophaga Human bites may result in infection with Eikenella corrodens, a gram-negative bacillus that is part of the normal flora of the human mouth. This infection can follow fist fights with blows to the mouth 75) Which of the following infections is commonly known to occur from contact with infected sheep? A. Oculoglandular syndrome of Parinaud B. Infectious pustular dermatitis C. Cutaneous anthrax D. Infectious pustular dermatitis and cutaneous anthraxCorrect Choice E. Erysipeloid Infectious pustular dermatitis is another name for orf, or ecthyma contagiosum. It is transmitted to humans by a parapoxvirus in sheep, goats, and reindeer. Cutaneous anthrax is also called "woolsorter's disease" and is caused by Bacillus anthracis, a gram-positive rod transmitted from contact with carcasses of infected sheep, cows, goats, and horses. The oculoglandular syndrome of Parinaud is the combination of granulomatous conjunctivitis and preauricular lymphadenopathy seen in cat-scratch disease 76) A goat farmer presents with fever and flu-like symptoms, as well as violaceous papulonodules on the trunk and lower extremities. You consider Malta fever in your differential diagnosis. Which of the following statements regarding this diagnosis is correct? A. Skin lesions are infrequently seen B. None of these answers are correct C. All of these answers are correctCorrect Choice D. Treatment is with rifampin E. Treatment is with doxycycline Malta fever is another name for Brucellosis. This infection is caused by ingesting raw goat milk and unpasteurized goat cheese infected with Brucellae (gram-negative rods). It is infrequent in the US, occurring mainly in veterinarians and farmers. Acute brucellosis has very non-specific features and presents as a flu-like illness. Skin lesions are infrequent (20% of cases) but are usually violaceous papulonodules on the trunk and lower extremities. Treatment is with doxycycline or rifampin 77) Which disease is transmitted by this insect? A. Visceral leishmaniasis B. African sleeping sickness C. OnchocerciasisCorrect Choice D. Chagas disease E. Dracunculiasis The insect depicted is the black fly, Simulium species. The black fly is the vector for onchocerciasis, which is caused by onchocerca volvulus. Clinical manifestation of this disease include atrophy, depigmentation and thickening of the skin. In addition, the infection may cause "river blindess" and onchocercomas, which are nodules containing microfilariae 78) This organism is: 20
  • 21. A. Chigoe flea B. Pthirus pubis Correct Choice C. Pediculus capitis D. Pulex irritans E. Pediculus humanus This is a crab louse. It is shorter and wider than the head and body louse. Note the crab-like claws 79) A 54 year-old man with psoriatic arthritis on methotrexate and infliximab calls you after he is bitten deeply on the ankle by his daughter's 2 year-old Chihuahua. He is concerned that his immunocompromised status puts him at risk for infection and potential complications from this bite. You correctly tell him: A. Capnocytophaga from dog bites can cause sepsis in immunocompromised victimsCorrect Choice B. As long as the dog has been vaccinated, he need not worry about serious infections C. Eikenella corrodens is the most common cause of infection following dog bites D. Pasteurella multocida infection is frequent after dog bites but not cat bites E. Rickettsial pox can be seen after dog bites Local infection and cellulitis is a concern following domesticated animal bites; vaccination may prevent against rabies, but not other infections. Cat bites most frequently result in Pasteurella multocida infections, while dog bites result in Pasteurella canis. However, most bites are polymicrobial with mixed aerobes and anaerobes. Infrequently systemic infections may result in brain or lung abscess, endocarditis, or sepsis. Sepsis is more frequent in immunocompromised victims and may be due to Capnocytophaga or Pasteurella 80) What is the initial treatment of choice for this dermatitis? A. Warm compresses B. Topical steroid C. Topical antibiotics D. 5% acetic acidCorrect Choice E. Observation The linear appearance of the dermatitis is typical of jellyfish stings. To prevent further nematocyte activation, 5% acetic acids should be applied for 15-30 minutes. Papain found in meat tenderizers or 70% isopropyl alcohol may also be applied. Warm compresses are contraindicated as they may increase systemic absorption of the venom 81) Pineapples contain which of the following proteolytic enzymes? A. None of these answers are correct B. BromelinCorrect Choice C. Phorbol esters D. Ranunculin E. All of these answers are correct 21
  • 22. Bromelin is a proteolytic enzyme found in pineapples. It is granted access to dermal vessels by calcium oxalate-induced fissures. Phorbol esters are found in plants of the Euphorbiaceae family. Ranunculin is an irritant found in buttercups (Ranunculaceae family 82) Which of the following infections is caused by a gram-positive rod? A. Brucellosis B. Erysipeloid of RosenbachCorrect Choice C. Cat-scratch disease D. Milker's nodule E. Glanders Erysipeloid of Rosenbach is an infection with Erysipelothrix insidiosa or Erysipelothrix rhusiopathiae, gram-positive rods. Brucellosis is caused by infection with Brucellae (gram-negative rod). Cat- scratch disease is caused by infection with Bartonella henselae, a gram-negative rod. Glanders is caused by infection with Pseudomonas mallei, a gram-negative rod. Milker's nodule is caused by the paravaccinia viurs, a parapoxvirus related to the orf virus 83) Which of the following vectors transmits Borrelia duttonii? A. Rhipicephalus sanguineus B. Amblyomma americanum C. Ornithodoros moubataCorrect Choice D. Ixodes pacificus E. none of the above Borrelia duttonii is a pathogenic organism of relapsing fever. It is transmitted by Ornithodoros moubata. Ixodes pacificus is one of the vectors of Lyme Borreliosis. Rhipicephalus sanguineus is a vector of Ehrlichiosis. Amblyomma americanum is a vector of Lyme Borreliosis and Rocky Mountain Spotted Fever 84) Ehrlichiosis is classified in two primary forms. These are: A. Intracellular and extracellular B. Primary and secondary C. None of these answers are correct D. Endemic and epidemic E. Monocytic and granulocyticCorrect Choice Ehrlichiosis comes in two forms: monocytic, which invades mononuclear WBCs, and granulocytic, in which peripheral PMNs show intracytoplasmic inclusions. Patients present with fever, chills, headache, and myalgias, along with leukopenia, anemia, and thrombocytopenia 85) Capsaicin causes which of the following? A. Blocking the influx of sodium ions into the cell B. Decreased firing of nerve fibers transmitting pain C. Increased substance P release Correct Choice 22
  • 23. D. Creating a barrier between nerves and their stimuli E. Decreased substance P release Repeated substance P release leads to eventual depletion of this agent which causes pain transmission 86) Which of the following infections is caused by a gram-negative rod? A. Bacillary angiomatosis B. Malta fever C. None of these answers are correct D. Farcy E. All of these answers are correctCorrect Choice Malta fever is another name for Brucellosis, an infection caused by Brucellae (gram-negative rod). Farcy is another name for Glanders, an infection with the gram-negative rod Pseudomonas mallei. Bacillary angiomatosis is caused by infection with Bartonella henselae, a gram-negative rod 87) Sesquiterpene lactones are found in which of the following plants? A. Alstromeria B. Pinus palustris C. Gingko D. ArtichokeCorrect Choice E. Chive Sesquiterpene lactones are sensitizers found in plants of the Asteraceae (Compositae) family. Members include ragweed, pyrethrum, chrysanthemum, weeds, feverfew, and artichoke. The sensitzer found in alstromeria (Peruvian lily) is tuliposide A (a glycoside). The Pinus palustris tree is the source of colophony. Chives contain diallyl disulfide. Gingko fruit pulp is a sensitizer 88) Match the photo to the plant identified: A. Compositae B. Alstromeria Correct Choice C. Primulaceae D. Anacardiacea E. Alliaceae Alstromeria is pictured 89) Which of the following contains active furocoumarins? A. Fig Correct Choice B. Lichens C. Mango D. Chrysanthemum 23
  • 24. E. Primrose The fig tree may cause a phytophotodermatitis 90) The necrosis and hemolysis that can result from a brown recluse spider bite may be attributed to which of the following toxins? A. Lipase B. Sphingomyelinase-DCorrect Choice C. Alpha lactrotoxin D. Hyaluronidase E. Histamine The brown recluse spider (Loxosceles reclusa) is a tan spider with a violin-shaped marking on the abdomen. Its bite may be painless, however the patient may develop erythema, a vesicle, and eventual necrosis. This may result in the "red, white, and blue sign." Systemic reactions (viscerocutaneous loxoscelism) may occur. The venom contains several toxins, however sphingomyelinase D seems to be causative of the necrosis and hemolysis 91) This patient acquired an infection with a microorganism that may have been inoculated into the skin from contact with: A. Grasses B. Rose thorns C. All of these answers are correctCorrect Choice D. Sphagnum moss E. None of these answers are correct This patient has sporotrichosis, which is caused by Sporothrix schenckii. Sporothrix schenckii can be inoculated into the skin by grasses, sphagnum moss, and rose thorns 92) If suspicious for scabies infestation, which of the following areas is least likely to be involved? A. External genitalia B. Umbilicus C. Fingerwebs D. Nipples E. Occipital scalpCorrect Choice Erythematous papules and burrows are commonly seen in webspaces, nipples, external genitalia, umbilicus and axilla in sarcoptes scabiei infestation 93) Which of the following statements regarding orf is correct? A. Orf is also known as "pseudocowpox." B. Lesions last indefinitely without treatment C. The papillomatous stage of lesion progression preceeds the regenerative stage D. The virus is very sturdy and survives many months in cold weatherCorrect Choice 24
  • 25. E. The papular stage of lesion progression follows the target stage Orf, also known as ecthyma contagiosum or infectious pustular dermatitis, is transmitted to humans by a parapoxvirus in sheep, goats, and reindeer. This is a self-limited condition with an excellent prognosis lasting approximately 6 weeks. Lesion morphology changes through a series of stages: papular (red, elevated) -> target (nodule with red center, white middle ring, red periphery) --> acute (red, weeping) --> regenerative (thin, dry crust overlying lesion) --> papillomatous --> regressive. The virus is very sturdy and survives many months in cold weather. Milker's nodule, caused by the paravaccinia virus (a parapoxvirus, related to the orf virus), infects cows and can be transmitted to dairy workers, and is also known as pseudocowpox 94) The majority of phytophotodermatitis is caused by plants from which family? A. ApiaceaeCorrect Choice B. Compositae C. Liliaceae D. Alliaceae E. None of these answers are correct The Apiaceae family (formerly Umbelliferae) includes parsley, celery, parsnip, hogweed, and fennel, and causes the majority of phytophotodermatitis 95) Thorns and barbs on blackberry bushes have been known to cause secondary infection through inoculation of: A. None of these answers are correct B. Mycobacterium ulcerans C. Sporothrix schenckii D. All of these answers are correct E. Mycobacterium kansaiiCorrect Choice Thorns and barbs on blackberry bushes have been known to cause secondary infection through inoculation of Mycobacterium kansaii. Sporothrix schenckii can be innoculated by grasses, sphagnum moss, and rose thorns. Mycobacterium ulcerans has been innoculated by spiky tropical vegetation 96) Match the photo to the plant identified: A. Toxicodendron B. Compositae Correct Choice C. Primulaceae D. Alstromeria E. Alliaceae This is ragweed, a member of the compositae (or asteraceae) family 97) Plants from the family Moraceae include which of the following? A. All of these answers are correct B. Fennel 25
  • 26. C. None of these answers are correct D. Burning bush E. Ficus caricaCorrect Choice Ficus carica (fig tree) is a member of the family Moraceae and a cause of phytophotodermatitis. Burning bush is a member of the family Rutaceae. Fennel is a member of the family Apiaceae 98) Identify this tick: A. Dermacentor andersoni B. Rhipicephalus sanguineus C. Dermacentor variabilis D. Amblyomma americanumCorrect Choice E. Ixodes dammini This is Amblyomma americanum which transmits Lyme as well as RMSF 99) Which of the following is a common cause of seabather's eruption? A. Linuche unguiculata B. Thimble jelly fish C. All of these answers are correctCorrect Choice D. Edwardsiella lineate E. Sea anemone Seabather's eruption (or sea lice) can be sporadic or part of an outbreak. Clinically, it appears as dermatitis beneath areas covered by swim suits. The geographic distribution is most frequent from the Caribbean to Bermuda, however there have been three outbreaks at Long Island, NY beaches. A variety of cnidarian larvae can cause this eruption, including thimble jelly fish (Linuche unguiculata) and sea anemone (Edwardsiella lineate). Quick removal of bathing suits and rinsing can help with prevention. Symptom relief is the mainstay of therapy 100) Phytophotodermatitis caused by skin exposure to limes is caused by: A. D-limonene B. Furocoumarin Correct Choice C. Tuliposide A D. Sesquiterpene lactone E. Ascorbic acid Phytophotodermatatitis (phototoxic eruptions from plants) are relatively commonly seen in florists, nursery personnel, gardeners, and farm workers. Plants that cause this eruption may produce photoxins such as furocoumarins Fig, fennel, lime, lemon, parsley, celery, dill, and hogweed are all causes of phytophotodermatitis 101) “Strimmer” dermatitis is caused by which of the following? A. M. marinum 26
  • 27. B. Lime C. Pseudomonas D. Cryptococcus E. Chervil Correct Choice Strimmer dermatitis is a phytophotodermatitis caused by spray of plant matter when using a weed- whacker or similar trimming tool 102) Identify this insect: A. Water flea B. Tsetse fly C. Mango fly D. Reduviid bugCorrect Choice E. Phlebotomous sandfly This a picture of the reduviid bug, also called the kissing bug or assassin bug. It is the vector for American Trypanosomiasis (Chagas disease), which is caused by Trypanosoma cruzii. The reduviid bug prefers to bite at mucocutaneous junctions 103) Solenopsin D is a: A. Phospholipase B. Piperdine derivative C. Hemolytic factor D. Hemolytic factor, piperdine derivative, histamine-releaserCorrect Choice E. Histamine-releaser Solenopsin D is a venom which is a hemolytic factor, a piperidine derative and leads to histamine release. The venom from honeybeeds contains phospholipase A 104) The majority of naturally-occurring cases of anthrax: A. Are extra-cutaneous B. Are acquired through ingestion of spores C. Are pulmonary D. Are cutaneousCorrect Choice E. Are oropharyngeal The majority (95%) of naturally-occurring cases of anthrax are of the cutaneous form, acquired from direct contact with the carcasses of dead sheep, cows, goats, and horses. Naturally-occurring pulmonary, gastrointestinal (acquired by ingestion of spores) and oropharyngeal infection with anthrax is less common 105) Ticks of the species shown: A. Can transit tularemia but not Rocky Mountain Spotted Fever B. Can transmit Rocky Mountain Spotted Fever and tularemiaCorrect Choice 27
  • 28. C. Can transmit Lyme borreliosis but not Rocky Mountain Spotted Fever or tularemia D. Can transmit Rocky Mountain Spotted Fever and Lyme borreliosis E. Can transmit Lyme borreliosis and tularemia The tick pictured is a male Dermacentor variabilis tick. Dermacentor variabilis can transmit Rocky Mountain Spotted Fever and tularemia. Lyme borreliosis is transmitted by Ixodes dammini, Ixodes pacificus, and Amblyomma americanum. 28